Download as pdf or txt
Download as pdf or txt
You are on page 1of 46

Topic Wise Distribution of Questions

Chapter Topic No of Questions


Making of the Constitution Constituent Assembly 4
Adoption 1
Fundamental Duties Fundamental Duties 3
Fundamental Rights Article 14 2
Article 21 2
Article 12 1
Article 30 2
Article 32 3
Article 13 1
Article 17 1
Article 18 1
Article 20 2
Articles 25-28 1
Article 19 1
Philosophy 2
Salient Features of Indian Federal System 2
Constitution Republic 2
Sources of Indian Constitution 2
Parliamentary System 3
Democracy 1
Secularism 1
Socialism 1
Parliamentary Sovereignty 1
Misc Chronological order of events 1
Lok Sabha Secretariat 1
Departments 1
Article 326 1
Article 300 1
Misc 18
Administrative Tribunals 1
Union Executive 1
Constitutional Amendment Constitutional Amendment 2
Union & Its Territory States formation 1
Preamble Preamble 3
Historical Background The Indian Councils Act, 1861 2
Government of India Act, 1919 1
Regulating Act 1773 1
Charter Act 1793 1
Charter Act 1813 1
DPSP 4
Directive Principles of State Gandhian 1
Policy
Schedules 2nd Schedule 1
6th Schedule 1
10th Schedule 1
5th Schedule 1
8th Schedule 1
9th Schedule 1
Governor Article 200 1
Mixed 1
Supreme Court Court of Record 1
Ad-hoc judge 1
Benches 1
NALSA 1
Original Jurisdiction 1
Judicial Review 1
Article 348 1
Parliament Sessions of Parliament 1
Union Council of Ministers 1
Money Bill 1
Status of a bill 1
Parliamentary proceedings 1
Lok Sabha member qualification 1
Chairman of Rajya Sabha 1
Disqualification of an MP 1
Leader of Opposition 1
Speaker 2
Cabinet 1
President’s Rule 1
Motions 3
Cabinet Secretary 1
Whip 1
Grants 1
Cabinet Committees 1
Pro-tem speaker 2
Finance Bill 1
Accountability 1
Cabinet Secretariat 1
Parliamentary Committees Committee on Public Accounts 1
Joint Committee on Offices of profit 2
Departmentally Related Standing 1
Committee
President Election 2
Ordinance 1
Tenure 1
Immunities 1

AMIGOS IAS 2
Pardoning Powers 1
Veto 1
Functions 1
Oath 1
State Legislature State Legislative Council 2
Dissolution of State Legislative 1
Assembly
Emergency Financial Emergency 1
National Emergency 1
Subordinate Courts District Courts 1
Gram Nyayalaya 1
Citizenship Citizenship 3
Vice-President of India Removal 1
Judicial Activism PIL 1
Basic Structure of Indian Basic Structure 1
Constitution
High Court Judges 1
Oath 1
TOTAL 150

AMIGOS IAS 3
APPSC GR-2 MAINS TEST 4 (11-05-24)

INDIAN CONSTITUTION

1. The Election Commission of India administers account in any scheme of reorganisation of


elections to the: states?
A. Lok Sabha A. Financial, Economic and Administrative.
B. Rajya Sabha B. Planning and promotion of social welfare.
C. State Legislative Assemblies in India C. Linguistic and cultural homogeneity.
D. The office of the President D. Preservation and the strengthening of
E. The office of Speaker of the Lok Sabha unity and security of the nation.
Select the correct answer using the codes Select the correct answer using the code given
given below: below:
(1) A, B and C (2) A, B, C and D (1) A and C (2) A and D
(3) A and C (4) All of the above (3) B, C and D (4) All of the above
ANSWER: 2 ANSWER: 4
EXPLANATION: EXPLANATION:
 The Election Commission of India is an Fazl Ali commission identified four major
autonomous constitutional authority factors that can be taken into account in any
responsible for administering Union and scheme of reorganisation of states:
State election processes in India. The body (a) Preservation and strengthening of the
administers elections to the Lok Sabha, unity and security of the country.
Rajya Sabha and State Legislative (b) Linguistic and cultural homogeneity.
Assemblies in India, and the offices of the (c) Financial, economic and administrative
President and Vice President in the considerations
country.  Planning and promotion of the welfare of
2. Arrange the following events in correct the people in each state as well as of the
chronological order: nation as a whole.
A. Formation of state of Himachal Pradesh 4. Consider the following statements:
B. Formation of state of Gujrat A. Zonal Councils are constitutional bodies.
C. Formation of state of Arunachal Pradesh B. Prime Minister acts as a common
D. Formation of state of Sikkim chairman of Zonal Councils
Select the correct answer using the code given Which of the statements given above is/are
below correct?
(1) B-D-A-C (2) A-B-D-C (1) Only A (2) Only B
(3) B-A-D-C (4) B-A-C-D (3) Both A and B (4) Neither A nor B
ANSWER: 3 ANSWER: 4
EXPLANATION: EXPLANATION:
• Gujrat was the 15th state of Indian Union  Zonal Councils are Statutory bodies, not
(1960) constitutional bodies. They are
• Himachal Pradesh: 17th state (1971) established by an Act of the Parliament
• Sikkim: 22nd state (1974) that is, States Reorganisation Act of 1956.
• Arunachal Pradesh: 24th state (1987) The Home Minister acts as a common
3. What were the factors identified by the Fazl chairman of Zonal Councils.
Ali commission that can be taken into

AMIGOS IAS 4
5. Consider the following statements: 7. Consider the following statements about the
A. The Chairman and members of a state powers given to Gram Sabha under
public service commission are appointed Panchayats Extension to Scheduled Areas
and removed by the Governor of the state. (PESA) Act:
B. The chairman and members of the Joint A. It promotes participatory democracy
State Public Service Commission are through Gram Sabha.
appointed by the President. B. Gram Sabha has the power of
Which of the statements given above is/are management of River Basins in that
correct? particular area.
(1) Only A (2) Only B Which of the statements given above is/are
(3) Both A and B (4) Neither A nor B correct?
ANSWER: 2 (1) Only A (2) Only B
EXPLANATION: (3) Both A and B (4) Neither A nor B
 The Chairman and members of a state ANSWER: 1
public service commission, though EXPLANATION:
appointed by the governor of the state,  One of its objectives is to have village
can be removed only by the President. governance with participatory democracy
 The Parliament can establish a Joint State and to make the gram Sabha a nucleus of
Public Service Commission (JSPSC) for two all activities.
or more states on the request of the state Powers given to Gram Sabha/ Panchayats at
legislatures concerned. The chairman and appropriate level under PESA Act:
members of the JSPSC are appointed by • Right to mandatory consultation in land
the president. acquisition, resettlement and
6. Which of the following are members of the rehabilitation of displaced persons,
appointment committee of the Director of • planning and management of minor water
Central Bureau of Investigation? bodies
A. The Leader of Opposition in the Lok Sabha • mandatory recommendations for
B. The Prime Minister prospective licenses/lease for mines and
C. The union minister of home affairs concession for the exploitation of minor
D. The Union minister of Law and Justice minerals
Select the correct answer using the code given • Ownership of minor forest produces etc.
below. 8. Which among the below mentioned
(1) A and B (2) A, B and C provision(s) ensures the independent and
(3) A, B and D (4) All of the above impartial functioning of the Election
Commission
ANSWER: 1
A. The chief election commissioner is
EXPLANATION:
provided with the security of tenure. He
 The CBI is headed by a Director. He is cannot be removed from his office except
assisted by a special director or an in the same manner and on the same
additional director. The Central grounds as a judge of the Supreme Court
Government shall appoint the Director of
B. The service conditions of the chief election
CBI on the recommendation of a three-
commissioner cannot be varied to his
member committee consisting of
disadvantage after his appointment
• The Prime Minister as Chairperson
C. Any other election commissioner or a
• The Leader of Opposition in the Lok Sabha regional commissioner cannot be
• The Chief Justice of India or Judge of the removed from office except on the
Supreme Court nominated by him recommendation of the chief election
commissioner

AMIGOS IAS 2
Choose the correct answers using the code C. Organizing the Lok Adalats comes under
given below the ambit of this authority.
(1) A and C (2) A and B Which of the statements given above is/are
(3) Only C (4) All of the above correct?
ANSWER: 4 (1) Only A (2) A and C
EXPLANATION: (3) Only B (4) All of the above
 The chief election commissioner is ANSWER: 2
provided with the security of tenure. He EXPLANATION:
cannot be removed from his office except  Article 39A of the Constitution of India
in same manner and on the same grounds provides for free legal aid to the poor and
as a judge of the Supreme Court. In other weaker sections of the society and
words, he can be removed by the ensures justice for all. It is one of the
president on the basis of a resolution objectives of NALSA.
passed to that effect by both the Houses  The National Legal Services Authority
of Parliament with special majority, either (NALSA) has been constituted under the
on the ground of proved misbehaviour or Legal Services Authorities Act, 1987 to
incapacity. Thus, he does not hold his monitor and evaluate implementation of
office till the pleasure of the president, legal aid programmes and to lay down
though he is appointed by him. policies and principles for making legal
 Article 324 of the Constitution to ensure services available under the Act.
the independent and impartial  NALSA lays down policies, principles,
functioning of the Election commission guidelines and frames effective and
has made the following provisions: The economical schemes for the State Legal
service conditions of the chief election Services Authorities to implement the
commissioner cannot be varied to his Legal Services Programmes throughout
disadvantage after his appointment.2. the country. Primarily, the State Legal
Any other election commissioner or a Services Authorities, District Legal
regional commissioner cannot be Services Authorities, Taluk Legal Services
removed from office except on the Committees, etc. have been asked to
recommendation of the chief election discharge the following main functions on
commissioner. Though the constitution a regular basis : 1. To provide free and
has sought to safeguard and ensure the competent legal services to the eligible
independence and impartiality of the persons. 2. To organize Lok Adalats for
Election Commission, some flaws can be amicable settlement of disputes. 3. To
noted, viz., 1. The Constitution has not organize legal awareness camps in the
prescribed the qualifications (legal, rural areas.
educational, administrative or judicial) of 10. Consider the following statements about the
the members of the Election Commission. 73rd amendment act of 1992:
2. The Constitution has not specified the
A. It gave a practical shape to the provision
term of the members of the Election
of the Directive Principles of the State
Commission. 3. The Constitution has not
Policy.
debarred the retiring election
commissioners from any further B. It provides for a three-tier system of
appointment by the government Panchayati raj in every state.
9. Consider the following statements about the C. It provided for the reservation of not less
National Legal Services Authority (NALSA): than one-third of the total number of
seats for women.
A. It is in consonance with the Directive
Principles of State Policy. Which of the statements given above is/are
correct?
B. It has been created under the executive
order of the Government of India. (1) Only A (2) A and C
(3) B and C (4) All of the above

AMIGOS IAS 3
ANSWER: 4
EXPLANATION: 12. Consider the following statements regarding
 The act has given a practical shape to Delimitation Commission in India:
Article 40 of the Constitution which says A. The orders issued by the Delimitation
that, ―The State shall take steps to Commission can be challenged only in
organise village panchayats and endow supreme court.
them with such powers and authority as B. The copies of its orders are laid before
may be necessary to enable them to both the house of parliament and the
function as units of self-government. This State Legislative Assembly concerned.
article forms a part of the Directive Which of the statements given above is/are
Principles of State Policy. correct?
 The act provides for a three-tier system of (1) Only A (2) Only B
Panchayati raj in every state, that is, (3) Both A and B (4) Neither A nor B
panchayats at the village, intermediate, ANSWER: 4
and district levels. Thus, the act brings
EXPLANATION:
about uniformity in the structure of
Panchayati raj throughout the country.  Delimitation literally means the act or
However, a state having a population not process of fixing limits or boundaries of
exceeding 20 lakh may not constitute territorial constituencies in a country or a
panchayats at the intermediate level. province having a legislative body. The job
of delimitation is assigned to a high-power
 The act provides for the reservation of not
body. The Delimitation Commission in
less than one-third of the total number of
India is a high-power body whose orders
seats for women (including the number of
have the force of law and cannot be called
seats reserved for women belonging the
in question before any court.
SCs and STs). Further, not less than one-
third of the total number of offices of  The copies of its orders are laid before the
chairpersons in the panchayats at each House of the People and the State
level shall be reserved for women. Legislative Assembly concerned, but no
modifications are permissible therein by
11. Consider the following commissions
them.
A. National Commission for Women
13. Consider the following pairs with reference
B. National Commission for Minorities to special status with respect to some states:
C. National Human Rights Commission Article State
D. National Commission for Protection of A. 371 A Nagaland
Child Rights
B. 371 B Arunachal Pradesh
Which among the commissions given above
C. 371 C Assam
is/are formed via a statute?
D. 371 D Manipur
(1) Only A (2) B and C
Which of the pairs given above is/are correctly
(3) A, C and D (4) All of the above
matched?
ANSWER: 4
(1) Only A (2) A, B and D
EXPLANATION:
(3) B and C (4) All of the above
 The national commissions like the
ANSWER: 1
National Commission for Women (1992),
EXPLANATION:
the National Commission for Minorities
(1993), the National Commission for  Article 371 A: Nagaland
Backward Classes (1993), the National  Article 371 B: Assam
Human Rights Commission (1993) and the  Article 371 C: Manipur
National Commission for Protection of  Article 371 D: Andhra Pradesh
Child Rights (2007) are statutory bodies in 14. Consider the following statements about the
the sense that they are established by acts Advocate General of the State:
of the Parliament.
AMIGOS IAS 4
A. He is appointed by a specially constituted  The Constitution makes a provision for the
committee headed by the Chief Minister. establishment of a Joint State Public
B. He has the right to vote in the proceedings Service Commission (JSPSC) for two or
of both the houses of state legislature. more states. While the UPSC and the SPSC
Which of the following statements given are created directly by the Constitution, a
above is/are incorrect? JSPSC can be created by an act of
(1) Only A (2) Only B Parliament on the request of the state
(3) Both A and B (4) Neither A nor B legislatures concerned. Thus, a JSPSC is a
statutory and not a constitutional body.
ANSWER: 3
 A JSPSC presents its annual performance
EXPLANATION:
report to each of the concerned state
 The advocate general is appointed by the governors. Each governor places the
governor. He must be a person who is report before the state legislature.
qualified to be appointed a judge of a high
 The chairman and members of a JSPSC are
court. In other words, he must be a citizen
appointed by the president. They hold
of India and must have held a judicial
office for a term of six years or until they
office for ten years or been an advocate of
attain the age of 62 years, whichever is
a high court for ten years.
earlier. They can be suspended or
 The term of office of the advocate general removed by the president. They can also
is not fixed by the Constitution. Further, resign from their offices at any time by
the Constitution does not contain the submitting their resignation letters to the
procedure and grounds for his removal. president.
He holds office during the pleasure of the
16. Consider the following statements about the
governor. This means that he may be
Union Public Service Commission:
removed by the governor at any time.
A. The conditions of service of the chairman
 He has the right to speak and to take part
and other members of the commission is
in the proceedings of both the Houses of
determined by the Parliament.
the state legislature or any committee of
B. Half of the members of the Commission
the state legislature of which he may be
should be persons who have held office
named a member, but without a right to
for at least ten years under the
vote. He enjoys all the privileges and
Government of India.
immunities that are available to a member
of the state legislature. Which of the statements given above is/are
correct?
15. Consider the following statements about the
Joint State Public Service Commission (1) Only A (2) Only B
(JSPSC): (3) Both A and B (4) Neither A nor B
A. It is a constitutional body created by the ANSWER: 2
President on the request of the state EXPLANATION:
legislatures concerned.  The Constitution authorises the President
B. A JSPSC presents its annual performance to determine the conditions of service of
report to the each of the concerned state the chairman and other members of the
governors. Union Public Service Commission.
C. The Chairman and its members can be  No qualifications are prescribed for the
removed by the UPSC. Commission‘s membership except that
Which of the statements given above is/are onehalf of the members of the
incorrect? Commission should be such persons who
(1) Only A (2) B and C have held office for at least ten years
(3) A and C (4) A and B either under the Government of India or
under the government of a state.
ANSWER: 3
17. Consider the following statements about the
EXPLANATION:
National Integration Council:

AMIGOS IAS 5
A. It was constituted during the process of president at any time. The remuneration
integration of princely states in 1948. of the AG is not fixed by the Constitution.
B. The union Home minister is the Chairman He receives such remuneration as the
of this council. president may determine.
Which of the statements given above is/are  The Attorney General is not a full-time
correct? counsel for the Government. He does not
(1) Only A (2) Only B fall in the category of government
(3) Both A and B (4) Neither A nor B servants. Further, he is not debarred from
ANSWER: 4 private legal practice.
EXPLANATION: 19. Consider the following statements about the
Comptroller and Auditor General of India:
 The National Integration Council (NIC) was
A. She/he has equal freedom with regard to
constituted in 1961, following a decision
both the audit of expenditure and also the
taken at a national conference on ‗unity
audit of receipts, stores and stock.
in diversity‘, convened by the Central
government, at New Delhi. It consisted of B. She/he can conduct the propriety audit.
the prime minister as chairman, central Which of the statements given above is/are
home minister, chief ministers of states, correct?
seven leaders of political parties, the (1) Only A (2) Only B
chairman of the UGC, two educationists, (3) Both A and B (4) Neither A nor B
the commissioner for SCs and STs and ANSWER: 2
seven other persons nominated by the EXPLANATION:
prime minister.  The CAG has more freedom with regard to
18. Consider the following statements about the audit of expenditure than with regard to
Attorney General of India: audit of receipts, stores and stock.
A. He must be a person who is qualified to be  In addition to this legal and regulatory
appointed a judge of the Supreme Court. audit, the CAG can also conduct the
B. His term of office and remuneration is propriety audit, that is, he can look into
fixed by the Constitution of India. the ‘wisdom, faithfulness and economy‘ of
C. As he is a Government servant, he is government expenditure and comment
debarred from private legal practice. on the wastefulness and extravagance of
Which of the statements given above is/are such expenditure.
correct? 20. Consider the following statements about the
(1) Only A (2) B and C Finance Commission:
(3) Only B (4) All of the above A. It is a quasi-judicial body.
ANSWER: 1 B. It is constituted by the Union Finance
EXPLANATION: Minister.
 The Constitution (Article 76) has provided C. Parliament determines the qualifications
for the office of the Attorney General for of members of the commission.
India. He is the highest law officer in the Which of the statements given above is/are
country. The Attorney General (AG) is correct?
appointed by the president. He must be a (1) Only A (2) Only B
person who is qualified to be appointed a (3) A and C (4) All of the above
judge of the Supreme Court. ANSWER: 3
 The term of office of the AG is not fixed by EXPLANATION:
the Constitution. Further, the Constitution  Article 280 of the Constitution of India
does not contain the procedure and provides for a Finance Commission as a
grounds for his removal. He holds office quasi-judicial body.
during the pleasure of the president. This
means that he may be removed by the

AMIGOS IAS 6
 It is constituted by the president of India B. The tenure, salaries and allowances are
every fifth year or at such earlier time as specified in the Constitution of India.
he considers necessary C. She/he submits the annual reports to the
 The Constitution authorises the President through the Union Minority
Parliament to determine the qualifications Affairs Minister.
of members of the commission and the Which of the statements given above is/are
manner in which they should be selected. correct?
21. Which of the committee/commission (1) Only A (2) B and C
recommended the deletion of Article 356? (3) Only C (4) All of the above
(1) Sarkari Commission ANSWER: 3
(2) Rajmannar Committee EXPLANATION:
(3) Administrative Reform Commission  She/he is to be appointed by the President
(4) MM Punchhi of India. It must be noted here that the
ANSWER: 2 Constitution does not specify the
EXPLANATION: qualifications, tenure, salaries and
 The Rajamannar Committee (1971) allowances, service conditions and
recommended the deletion of Articles 356 procedure for removal of the Special
and 357 from the Constitution of India. Officer for Linguistic Minorities.
The committee also recommended that  At the Central level, the Commissioner
the governor should not consider himself falls under the Ministry of Minority Affairs.
as an agent of the Centre, but instead play Hence, he submits the annual reports or
his role as the constitutional head of the other reports to the President through the
state. Union Minority Affairs Minister.
22. Consider the following statements with 24. Consider the following statements about the
respect to National Commission on Backward National Commission for STs:
Classes (NCBC): A. It is created under an executive act of the
A. 102nd Constitutional amendment Central Government.
provided NCBC constitutional status. B. The Chairperson of this Commission is
B. Tenure and service conditions of the appointed by the President.
members will be determined by the C. Both the Central the state governments
parliament. are required to consult the Commission on
Which of the statement above is/are correct? all major policy matters affecting the STs.
(1) Only A (2) Only B Which of the statements given above is/are
(3) Both A and B (4) Neither A nor B correct?
ANSWER: 1 (1) Only A (2) Only B
EXPLANATION: (3) B and C (4) All of the above
 102nd constitutional amendment act ANSWER: 3
granted constitutional status to NCBC and EXPLANATION:
entrusted with the powers to examine the  The National Commission for Scheduled
matters backword classes. Tribes (STs) is also a constitutional body in
 Under the Constitution Amendment Bill, the sense that it is directly established by
the NCBC will comprise of five members Article 338-A of the Constitution. The
appointed by the President. Their tenure National Commission for SCs and STs
and conditions of service will also be came into being consequent upon passing
decided by the President through rules. of the 65th Constitutional Amendment
23. Consider the following statements about the Act of 1990.
Special Officer for Linguistic Minorities:  It consists of a chairperson, a vice-
A. She/he is appointed by the Ministry of chairperson and three other members.
Minority Affairs. They are appointed by the President by

AMIGOS IAS 7
warrant under his hand and seal. Their shall be filled. It may also make any
conditions of service and tenure of office provision for the constitution of
are also determined by the President. committees in addition to the wards
 The Central government and the state committees.
governments are required to consult the 26. Consider the following statements about the
Commission on all major policy matters National Human Rights Commission:
affecting the STs. A. It is a statutory body.
25. Consider the following statements about the B. Only a person having judicial background
salient features of the 74th amendment act can be the Chairman of this Commission.
of 1992: C. It has all the powers of a civil court and its
A. Members of a municipality shall be proceedings have a judicial character.
elected directly by the people of the Which of the statements given above is/are
municipal area. correct?
B. The conduct of all elections to the (1) Only A (2) A and B
municipalities shall be vested in the state (3) B and C (4) All of the above
election commission. ANSWER: 4
C. The Governor may make provision with EXPLANATION:
respect to the composition and territorial
 The National Human Rights Commission is
area of a wards committee.
a statutory (and not a constitutional)
Which of the statements given above is/are
body. It was established in 1993 under a
correct?
legislation enacted by the Parliament,
(1) Only A (2) A and B namely, the Protection of Human Rights
(3) Only C (4) All of the above Act, 1993. This Act was amended in 2006.
ANSWER: 2  The commission is a multi-member body
EXPLANATION: consisting of a chairman and four
 All the members of a municipality shall be members. The chairman should be a
elected directly by the people of the retired chief justice of India, and members
municipal area. For this purpose, each should be serving or retired judges of the
municipal area shall be divided into Supreme Court, a serving or retired chief
territorial constituencies to be known as justice of a high court and two persons
wards. The state legislature may provide having knowledge or practical experience
the manner of election of the chairperson with respect to human rights.
of a municipality.  It is vested with the power to regulate its
 The superintendence, direction and own procedure. It has all the powers of a
control of the preparation of electoral civil court and its proceedings have a
rolls and the conduct of all elections to the judicial character. It may call for
municipalities shall be vested in the state information or report from the Central
election commission. The state legislature and state governments or any other
may make provision with respect to all authority subordinate thereto.
matters relating to elections to the 27. Consider the following statements about the
municipalities. Central Information Commission:
 There shall be constituted a wards A. It is an attached office to the Ministry of
committee, consisting of one or more Information and Broadcasting.
wards, within the territorial area of a B. The Commission does have suo-motu
municipality having population of three power.
lakh or more. The state legislature may C. The Commission submits its annual report
make provision with respect to the to the President.
composition and the territorial area of a Which of the statements given above is/are
wards committee and the manner in correct?
which the seats in a wards committee

AMIGOS IAS 8
(1) Only A (2) B and C  The Central Vigilance Commission was set
(3) Only B (4) All of the above up by the Government in February,1964
ANSWER: 3 on the recommendations of the
EXPLANATION: Committee on Prevention of Corruption,
headed by Shri K. Santhanam, to advise
 The Central Information Commission is a
and guide Central Government agencies in
high-powered independent body which
the field of vigilance.
inter alia looks into the complaints made
to it and decide the appeals. It entertains  The Central Government is required to
complaints and appeals pertaining to consult the CVC in making rules and
offices, financial institutions, public sector regulations governing the vigilance and
undertakings, etc., under the Central disciplinary matters relating to the
Government and the Union Territories. members of Central Services and All-India
Services
 On the powers and functions of the
Central Information Commission is that it 29. Consider the following statements regarding
the Commission can order inquiry into any Union-State Relations during Emergencies.
matter if there are reasonable grounds A. During a national emergency centre takes
(suomoto power). full control of state government but they
 The Commission submits an annual report are not suspended.
to the Central Government on the B. During a financial emergency, the
implementation of the provisions of this governor of the state can give direction for
Act. The Central Government places this the reduction of salaries of the high court
report before each House of Parliament. judges.
28. Consider the following statements about the Which of the statements given above is/are
Central Vigilance Commission (CVC): correct?
A. It is a non-statutory and non- (1) Only A (2) Only B
constitutional body. (3) Both A and B (4) Neither A nor B
B. It was set up by the Government on the ANSWER: 1
recommendations of the K. Santhanam EXPLANATION:
Committee on Prevention of Corruption.  During the operation of a national
C. It is consulted by the Central Government emergency (under Article 352), the Centre
on disciplinary matters relating to the becomes entitled to give executive
members of All-India Services. directions to a state on ‘any’ matter. Thus,
Which of the statements given above is/are the state governments are brought under
correct? the complete control of the Centre,
(1) Only A (2) B and C though they are not suspended.
(3) Only C (4) All of the above  During the operation of a financial
ANSWER: 2 emergency (under Article 360), the Centre
EXPLANATION: can direct the states to observe canons of
financial propriety and the President can
 The Central Vigilance Commission (CVC) is
give other necessary directions including
the main agency for preventing corruption
the reduction of salaries of persons
in the Central government. It was
serving in the state and the high court
established in 1964 by an executive
judges
resolution of the Central government.
30. In the context of administration of Union
Originally the CVC was neither a
Territories, which of the following
constitutional body nor a statutory body.
statements is not correct?
Later, in 2003, the Parliament enacted a
law conferring statutory status on the (1) Administrator of Union Territory is an
CVC. agent of the President and not head of
state

AMIGOS IAS 9
(2) Puducherry and Delhi assemblies are the matter to President and seek
empowered to make laws on State List but directions and act accordingly
not on Central List and Concurrent List (4) The union territories of Delhi and
(3) Constitutional provisions for Chandigarh share same High Court
administration of Union Territories also ANSWER: 3
apply to acquired territories EXPLANATION:
(4) There is no uniformity in the  The 69th Constitutional amendment act of
administrative system of Union Territories 1991provided a special status to the
ANSWER: 2 Union Territory of Delhi, and redesignated
EXPLANATION: it as the National Capital Territory of Delhi
 Articles 239 to 241 of the Constitution and designated the administrator of Delhi
deal with the Union territories. Even as the lieutenant Governor. The strength
though all the Union territories belong to of the assembly is fixed at 70 members,
one category, there is no uniformity in directly elected by the people.
their administrative system.  The strength of the council of ministers is
 Every union territory is administered by fixed at ten percent of the total strength
the President acting through an of the assembly, that is, seven- one chief
administrator appointed by him. An minister and six other ministers.
administrator of a union territory is n  The council of ministers headed by the
agent of the President and not head of chief minister aid and advises the it
state like Governors. governor in the exercise of his functions
 The Parliament can make laws on any except in so far as he is required to act in
subject of the three lists (Union list, his discretion. In the case of difference of
Concurrent list and State List) for the opinion between the
Union territories. This power of  It governor and his ministers, the it
Parliament also extends to Puducherry governor is to refer the matter to the
and Delhi, which have their own local President for decision and act accordingly.
legislatures. This means that, the 32. Which of the following ministries is the nodal
legislative power of Parliament for the ministry for all matters of Union Territories?
union territories on subjects of State List (1) Ministry of Home Affairs
remain unaffected even after the (2) Ministry of Personnel and Public
establishing a local legislature for them. Administration
But, the legislative assembly of
(3) Ministry of Urban Development
Puducherry can also make laws on any
subject of the State list and concurrent (4) Ministry of Defence
list. Similarly, the legislative assembly of ANSWER: 1
Delhi can make laws on any subject of the EXPLANATION:
State List (except public order, police and  The Union territories division in Ministry
land) and the Concurrent List. of Home Affairs deals with all legislative
31. With respect to National Capital Territory of and constitutional matters relating to
Delhi, which of the following statements is Union territories, including National
correct? Capital Territory of Delhi. It also functions
(1) Strength of the legislative assembly of as the cadre controlling authority of the
Delhi is 70 and the maximum size of Arunachal Pradesh-Goa-Mizoram-Union
council of ministers is 10 Territories (AGMUT) cadre of the Indian
(2) Lt. governor needs President’s permission Administrative Service (IAS), the Indian
only for an ordinance’s promulgation but Forest Service (IFS/IFoS), and the Indian
not withdrawal Police Service (IPS) as also Delhi, Andaman
and Nicobar Islands Civil Service
(3) In case of a difference of opinion between
(DANICS)/Delhi, Andaman and Nicobar
the ministers, the Lt. Governor must refer
Islands Police Service (DANIPS). Besides, it
AMIGOS IAS 10
is responsible for overseeing the crime  Entry 5 in State List: Local government,
and law and order situation in UTs. that is to say, the constitution and powers
33. Which among the following is/are voluntary of municipal corporations, improvement
provision(s) with regard to Local Self trusts, district boards, mining settlement
Governments? authorities and other local authorities for
A. Devolution of powers and responsibilities the purpose of local self-government or
upon Panchayats to perform some or all of village administration.
the 29 functions listed in XI Schedule of 35. “This type of urban government is
the Constitution established by the large public enterprises to
B. Granting powers and authority to provide civic amenities to its staff and
panchayats to enable them to function as workers living in housing colonies built near
institutions of self-government the plant.”
C. Providing reservation of l/3rd of seats for The above statement defines what type of
women in Panchayats at all three levels Local Government?
D. Indirect elections to the post of (1) Township
Chairperson of Panchayats at (2) Town Area Committee
intermediate and district levels (3) Notified Area Committee
Select the correct answer using the codes (4) Port Trust
given below ANSWER: 1
(1) B and D (2) A and D EXPLANATION:
(3) A, B and D (4) A and B  Township - PSUs are set up by the govt,
ANSWER: 4 and housing colonies have been set up
EXPLANATION: around them for the staff and workers.
 Devolution of powers and responsibilities These draw people from rural as well as
upon panchayats to prepare plans for urban areas and this develops into a kind
economic development and social justice; of a very small town, therefore it is named
and to perform some or all of the 29 as a Township. These townships are
functions listed in the Eleventh Schedule administered by the Municipal
of the Constitution. corporation or Municipality under which it
 Giving representation to members of the falls which appoints a Town Administrator
Parliament (both the Houses) and the for this area who is assisted by a few
state legislature (both the Houses) in the engineers and technicians that handle
panchayats at different levels falling functions like water, electricity, roads,
within their constituencies. Providing drainage, markets, parks, etc. The
reservation of seats (both members and expenditure on such townships are shared
chairpersons) for backward classes in equally by the urban local govt, as well as
panchayats at any level. the respective Industry.
 Granting financial powers to the  Town Area Committee - It is a semi
panchayats, i.e. authorizing them to levy, municipal authority constituted for small
collect and appropriate taxes, duties, tolls towns, and it exists in several states out of
and fees. which Uttar Pradesh has the largest
number. The members may be partly
34. Under which list of the VII schedule of the
elected and partly nominated by the state
Constitution, the subject of ‘Local
govt, or wholly nominated or wholly
Government’ is placed?
elected. It is assigned a number of
(1) Central List (2) State List
functions like street lighting, drainage,
(3) Concurrent List (4) None of the above roads, conservancy, etc. The District
ANSWER: 2 Collector in some states has been given
EXPLANATION: powers of surveillance and control over
the TAC.

AMIGOS IAS 11
 Notified Area Committee - In urban shall, as soon as may be within one year
planning, a Notified area is any land area from the commencement of the
earmarked by legal provision for future Constitution (Seventy-third Amendment)
development. Act, 1992, and thereafter at the expiration
36. Which among the following is not an area of every fifth year, constitute a Finance
based urban body? Commission to review the financial
(1) Cantonment Board position of the Panchayats and to make
(2) Port Trust recommendations to the Governor as to
(3) Industrial Corridor Board A. The principles which should govern
(4) Special Purpose Agency 1. The distribution between the State and
the Panchayats of the net proceeds of the
ANSWER: 4
taxes, duties, tolls and fees leviable by the
EXPLANATION: State, which may be divided between
 The states have set up Special Purpose them under this Part and the allocation
Agencies to undertake designated between the Panchayats at all levels of
activities or specific functions. These their respective shares of such proceeds;
functions actually belong to the domain of 2. The determination of the taxes, duties,
municipal corporations or municipalities tolls and fees which may be assigned as, or
or other local urban governments. Thus, appropriated by, the Panchayats;
these are function-based agencies and are
3. The grants-in-aid to the Panchayats from
known as 'single purpose', 'uni-purpose'
the Consolidated Fund of the State
or 'special purpose' agencies or 'functional
B. The measures needed to improve the
local bodies'.
financial position of the Panchayats;
Some of them are –
C. Any other matter referred to the Finance
1. Town improvement trusts.
Commission by the Governor in the
2. Urban development authorities. interests of sound finance of the
3. Water supply and sewerage boards. Panchayats.
4. Housing boards. 38. In which of the following areas, the PESA Act,
5. Pollution control boards. 1996 is implemented?
6. Electricity supply boards. (1) Fifth Schedule Areas
7. City transport boards. (2) Sixth Schedule Areas
37. Which of the following is correct regarding (3) Both a and b
State Finance Commissions? (4) Tribal-majority states only
(1) It is constituted by the President of India ANSWER: 1
every five years on recommendation of EXPLANATION:
the Governor of a State
 The Provisions of the Panchayats
(2) It lays down the principles that should (Extension to Scheduled Areas) Act, 1996
govern the distribution of net proceeds of or PESA is a law enacted by the
taxes, duties etc., between the centre, Government of India for ensuring self-
states and panchayats governance through traditional Gram
(3) The state legislature provides for the Sabhas for people living in the Scheduled
composition of the commission and Areas of India. Scheduled Areas are areas
qualification of its members identified by the Fifth Schedule of the
(4) It must take recommendations from Constitution of India. Scheduled Areas are
Central Finance Commission to augment found in ten states of India which have
the consolidated fund of a state predominant population of tribal
ANSWER: 3 communities.
EXPLANATION:  The Scheduled Areas, were not covered by
 Article 2431 of the Indian Constitution the 73rd Constitutional Amendment or
prescribes that the Governor of a State Panchayati Raj Act of the Indian

AMIGOS IAS 12
Constitution as provided in the Part IX of 40. Who among the following has powers to
the Constitution. PESA was enacted on 24 declare an area to be scheduled area?
December 1996 to extend the provisions (1) The President
of Part IX of the Constitution to Scheduled (2) Ministry of Home Affairs
Areas, with certain exceptions and (3) The Governor
modifications.
(4) National Commission for Scheduled Tribes
39. Which of the following statements is correct
ANSWER: 1
regarding Local Self Government in India?
EXPLANATION:
(1) Elections to Panchayats are conducted by
the Election Commission of India  As per the Constitutional provision under
Article 244 (1) of the Constitution of India,
(2) The gram Sabha consists of persons only
the 'Scheduled Areas' are defined as 'such
registered in the electoral rolls of a village
areas as the President may by order
within the panchayat
declare to be Scheduled Areas' -as per
(3) Panchayats cannot be dissolved before it paragraph 6(1) of the Fifth Schedule of the
completes 5-year term Constitution of India. The specification of
(4) The Panchayati raj system eliminates "Scheduled Areas" in relation to a State is
representative democracy to provide by a notified order of the President, after
participatory democracy consultation with the Governor of that
ANSWER: 2 State.
EXPLANATION:  In accordance with the provisions of
 "Gram Sabha" means a body consisting of paragraph 6(2) of the Fifth Schedule of the
persons registered in the electoral rolls Constitution of India, the President may
relating to a village comprised within the increase the area of any Scheduled Area in
area of Panchayat at the village level. a State after consultation with the
 Panchayats shall have a uniform five-year Governor of that State; and make fresh
term and elections to constitute new orders redefining the areas which are to
bodies shall be completed before the be Scheduled Areas in relation to any
expiry of term. In the event of dissolution, State. The same applies in the case of any
election will be compulsorily held within alteration, increase, decrease,
six months. A Panchayat constituted upon incorporation of new areas, or rescinding
the dissolution of a Panchayat before the any Orders relating to "Scheduled Areas"
expiration of its duration shall continue  At present, Scheduled Areas have been
only for the remainder of the period for declared in the States of Andhra Pradesh
which the dissolved Panchayat would (including Telangana), Chhattisgarh,
have continued. A person who is Gujarat, Himachal Pradesh, Jharkhand,
disqualified under any law, election to the Madhya Pradesh, Maharashtra, Odisha
legislature of the state or under any of the and Rajasthan
state will not be entitled to become a 41. Which of the following is correct regarding
member of a panchayat. provisions related to Languages in Indian
 Independent election commission will be Constitution?
established in the state to (1) The official languages of the Union are
superintendence, direction, and control of Hindi, Urdu and English
the electoral process and preparation of (2) Originally English and Urdu language was
electoral rolls. Specific responsibilities will provided for 25 years from the
be entrusted to the panchayats to prepare commencement of the Constitution
plans for economic development and (3) The President has the power to direct a
social justice in respect of matters listed in state to make a language to be officially
XI Schedule. recognised in that state

AMIGOS IAS 13
(4) Choice for official language of a State is (3) Supreme Court
limited to the languages in Eighth (4) Rajya Sabha
Schedule of the Constitution ANSWER: 1
ANSWER: 3 EXPLANATION:
EXPLANATION:  The Central Information Commission has
Official Language of the Union: been constituted with effect from 12-10-
 The official language of the Union shall be 2005 under the Right to Information Act,
Hindi in Devanagari script. The form of 2005. The jurisdiction of the Commission
numerals to be used for the official extends over all Central Public Authorities.
purposes of the Union shall be the The Commission has certain powers and
international form of Indian numerals. functions mentioned in sections 18, 19, 20
Notwithstanding anything, for a period of and 25 of the RTI Act, 2005.These broadly
fifteen years from the commencement of relate to adjudication in second appeal for
this Constitution, the English language giving information; direction for record
shall continue to be used for all the official keeping, suo moto disclosures receiving
purposes of the Union for which it was and enquiring into a complaint on inability
being used immediately before such to file RTI etc; imposition of penalties and
commencement: Provided that the Monitoring and Reporting including
President may, during the mentioned preparation of an Annual Report. The
period, by order authorise the use of the decisions of the Commission are final and
Hindi language in addition to the English binding. The Commission includes 1 Chief
language and of the Devanagari form of Information Commissioner (CIC) and not
numerals in addition to the international more than 10 Information Commissioners
form of Indian numerals for any of the (IC) who are appointed by the President of
official purposes of the Union. India.
Notwithstanding anything in this article, 43. Consider the following statements with
Parliament may by law provide for the reference to the administration of tribal
use, after the said period of fifteen years, areas:
of the English language, or the Devanagari A. The President is empowered to organize
form of numerals, for such purposes as and reorganise the autonomous districts
may be specified in the law. constituted in the tribal areas.
 Official Language or Languages of a State: B. The autonomous districts do not fall
Subject to the provisions of articles 346 outside the executive authority of the
and 347, the Legislature of a State may by state concerned
law adopt any one or more of the Which of the statements given above is/ are
languages in use in the State or Hindi as correct?
the language or languages to be used for
(1) Only A (2) Only B
all or any of the official purposes of that
(3) Both A and B (4) Neither A nor B
State: Provided that, until the Legislature
of the State otherwise provides by law, ANSWER: 2
the English language shall continue to be EXPLANATION:
used for those official purposes within the • The Sixth Schedule of the constitution
State for which it was being used provides special provisions for the
immediately before the commencement administration of the tribal areas in
of this Constitution. Assam, Meghalaya, Tripura and Mizoram.
42. Central Information Commissioners are The provisions of the schedule are as
appointed by which of the following follows:
authorities? • It provides for autonomous districts and
(1) President autonomous regions. But they do not fall
(2) Appointments Committee of Cabinet outside the executive authority of the
state concerned.

AMIGOS IAS 14
• The governor can increase, decrease, re- regions or apply with specified exceptions
organise or alter the boundary of these and modifications.
districts. • If at any time the Governor is satisfied that
• If there are different scheduled tribes in an act or resolution of a district or a
an autonomous district, the governor may regional council is likely to endanger the
divide the area or areas inhabited by them safety of India he may suspend such an act
into autonomous regions. or resolution and take such steps.
• Each autonomous district has a district 44. Division of subject of legislation under three
council consisting of not more than thirty legislation list is provided in:
members, of whom not more than four (1) Schedule 3 (2) Schedule 6
persons shall be nominated by the (3) Schedule 7 (4) Schedule 9
governor and the rest shall be elected on ANSWER: 3
the basis of adult suffrage. The elected
EXPLANATION:
members of the district council hold office
for a term of five years and nominated  The Seventh Schedule of the Indian
member hold office at the pleasure of the Constitution is one of 12 schedules that
governor. regulate the relationship between the
Union and State governments. It specifies
• The district and the regional councils can
the division of powers and functions
make laws on matters such as land,
between the two.
forests, canal water, Jhum cultivation,
local administration, inheritance of 45. Who among the following is the chairman of
property etc. the standing committee of the Inter-State
Council?
• The district and the regional councils may
constitute village councils or courts for the (1) The President of India
trial of suits and cases between the parties (2) The Prime Minister
all of whom belong to Scheduled Tribes (3) Union Home Minister
within such areas. (4) The Vice President of India
• The district council can establish, ANSWER: 3
construct, or manage primary schools, EXPLANATION:
dispensaries, markets, ferries, fisheries,  The Government has reconstituted the
roads, road transport and waterways in Inter State Council (ISC) and the Standing
the district. Committee of the Inter State Council
• A district fund for each autonomous under Clause 2 of the Inter State Council
district, and a regional fund for each order,1990
autonomous region is constituted to  The standing committee consists Union
which money received respectively by the Home minister as Chairman, 5 members
district council of Cabinet rank and 9 members Chief
• The district and the regional councils have Ministers of state
powers to assess and collect land revenue The Standing Committee will:
and to impose certain taxes. (i) have continuous consultation and process
• The district council is empowered to make matters for consideration of the Council
regulations for the Control of money- (ii) Process all matters pertaining to Centre
lending and trading by non-tribals State Relations before they are taken up
• Estimated receipts and expenditure for consideration in the inter-State
pertaining to autonomous districts has to Council
be shown separately in the annual (iii) Monitor the implementation of decisions
financial statement of the state. taken on the recommendations of the
• An act of parliament or of the legislature Council and
of the state does not apply to a (iv) Consider any other matter referred to it by
autonomous districts and autonomous the Chairman/Council.

AMIGOS IAS 15
(v) The Standing Committee may, if • the conduct of its meetings and its
necessary, invite experts and persons procedure in general; and
eminent in specific fields to have the • all other incidental matters.
benefit of their views while deliberation 47. The institution of Lokayukta was established
upon the related subjects first in which among the following states?
46. Consider the following statements with (1) Rajasthan (2) Kerala
reference to Tribal Advisory Council: (3) West Bengal (4) Maharashtra
A. It will be established on the direction of ANSWER: 4
the President only in a states having
EXPLANATION:
scheduled areas.
 Maharashtra was the first state to
B. The council have only advisory powers to
introduce the institution of Lokayukta
the state government.
through The Lokayukta and Upa-
C. The three-fourths of the members of the Lokayuktas Act in 1971.This was followed
council must be representatives of by similar acts that were enacted by the
Scheduled Tribes. states of Odisha, Rajasthan, Bihar, Uttar
Which of the statements given above is/ are Pradesh, Karnataka, Madhya Pradesh,
correct? Andhra Pradesh, Gujarat, Kerala, Tamil
(1) A and B (2) B and C Nadu and the union territory of Delhi.
(3) A and C (4) All of the above 48. Which one of the following states is the first
ANSWER: 2 state in India to operationalise a social audit
EXPLANATION: law?
 Para 4 of the Fifth Schedule provides for (1) Meghalaya (2) Maharashtra
establishment of a Tribes Advisory Council (3) Telangana (4) Kerala
(TAC) in any State having Scheduled Areas. ANSWER: 1
If the President so directs, there will be EXPLANATION:
established a TAC in a State having  Meghalaya became the first state in India
Scheduled tribes but not Scheduled Areas to operationalise a law that makes social
therein, consisting of not more than audit of government programmes and
twenty members of whom, three-fourths schemes a part of government practice
shall be the representatives of the
49. Consider the following statements with
Scheduled Tribes in the Legislative
reference to State Flag:
Assembly of the State. If the number of
representatives of the STs in the A. There is no prohibition in the constitution
Legislative Assembly of the State is less for state to have its own flag.
than the number of seats in the TAC to be B. The manner in which the state flag is
filled by such representatives, the hoisted should be always below the
remaining seats shall be filled by other national flag
members of those Tribes Which of the statements given above is/are
 The TAC shall advise on such matters correct?
pertaining to the welfare and the (1) Only A (2) Only B
advancement of the STs in the State as (3) Both A and B (4) Neither A nor B
may be referred to them by the Governor. ANSWER: 3
 The Governor may make rules prescribing EXPLANATION:
or regulating  Under the Constitution, a flag is not
• The number of members of the Council, enumerated in the Seventh Schedule.
the mode of their appointment and the However, Article 51A ordains that every
appointment of the Chairman of the citizen shall abide by the Constitution and
Council and of the officers and servants respect its ideals and institutions, the
thereof, national flag, and the national anthem.
There is no other provision regulating

AMIGOS IAS 16
hoisting of flags, either by the States or by is set up under the Cantonments Act of
the public. It is clear that there is no 1924—a legislation enacted by the Central
prohibition under the Constitution to Government.
hoist any flag other than the national flag.  In addition to the area- based urban
 Even the Flag Code of India, 2002 does not bodies (or multipurpose agencies, that is,
impose prohibitions on a State flag. On the municipal corporations, municipalities,
contrary, in the provisions regarding notified area committees, town area
hoisting of the national flag by the general committees, cantonment boards,
public, private organisations, educational townships and port trusts), the state
institutions, etc., the Code expressly governments have set up certain agencies
authorises the flying of other flags under to undertake designated activities or
the condition that they should not be specific functions which legitimately,
hoisted from the same masthead as the belong to the domain of municipal
national flag or placed higher than it. corporations or municipalities or other
50. Which of the following types of urban local urban governments.
government established by the Act of  A port trust is created by an Act of
Parliament? Parliament. It consists of both elected and
A. Notified Area Committee nominated members. Its chairman is an
B. Cantonment board official. Its civic functions are more or less
C. Township similar to those of a Municipality.
D. Special purpose agency 51. The institution of Panchayati Raj was
E. Port Trust inaugurated by Pandit Jawaharlal Nehru on
Rajasthan.
Select the correct answer using the codes
given below (1) 14th July, 1961 (2) 23rd April, 1965
(1) A, B and D (2) B and E (3) 2nd October, 1959 (4) 17th June, 1960
(3) A, C and D (4) All of the above ANSWER: 3
ANSWER: 2 EXPLANATION:
EXPLANATION:  Panchayat Raj system or local self-
governing institutions were inaugurated
 A notified area committee is created for
by Jawaharlal Nehru on 2 October 1959 at
the administration of two types of areas—
Nagaur in Rajasthan.
a fast developing town due to
industrialisation and a town which does 52. Consider the following statements with
not yet fulfill all the conditions necessary reference to official languages:
for the constitution of a municipality, but A. The Part XVI of the constitution of India
which otherwise are considered deals with the official languages.
important by the state government. Since B. Parliament has not made any provision for
it is established by a notification in the the use of Hindi in the Supreme Court.
government gazette, it called as notified C. The constitution imposes a duty upon the
area committee. Though It functions centre to promote the spread and
within the framework of the State development of Hindi language.
Municipal Act, only these provision of the Which of the statements given above is/ are
Act apply to it which are notified in the correct?
government gazette by which it is created. (1) A and B (2) B and C
It may also be entrusted to exercise (3) Only C (4) All of the above
powers under any other act. ANSWER: 2
 A cantonment board is established for EXPLANATION:
municipal administration for civilian
 The official language of the Union shall be
population in the cantonment area. It is a
Hindi in Devanagari script. The form of
delimited area where the military forces
numerals to be used for the official
and troops are permanently stationed. It

AMIGOS IAS 17
purposes of the Union shall be the Persons having special knowledge in municipal
international form of Indian numerals. administration.
 Notwithstanding anything, for a period of • Members of Lok Sabha and state
fifteen years from the commencement of legislature assembly representing the
this Constitution, the English language constituencies that comprise the
shall continue to be used for all the official municipal area.
purposes of the Union for which it was • Members of Rajya Sabha and the state
being used immediately before such legislative councils registered as electors
commencement: Provided that the within the municipal area.
President may, during the mentioned • The chair persons of the committees'
period, by order authorise the use of the other than wards committee.
Hindi language in addition to the English • Through nominated members and special
language and of the Devanagari form of invitees officers of district administration
numerals in addition to the international as well as members of the Central and
form of Indian numerals for any of the State legislatures find representation in
official purposes of the Union the DPC. Special invitees or permanent
 Notwithstanding anything in this article, invitees generally include local MPs and
Parliament may by law provide for the MLAs, district administration officials also
use, after the said period of fifteen years, representatives from line agencies like
of the English language, or the Devanagari District Cooperative Bank / Land
form of numerals, for such purposes as Development Banks. These invitees do not
may be specified in the law. have voting rights in the DPC but
 The Parliament has not made any law participate in its deliberations.
prescribing Hindi to be used as a language 54. Under which of the following ministries does
of Supreme Court, and hence the sole the Department of Official Language (Raj
language of Supreme Court has been Bhasha Vibhag) come?
English. Incidents have occurred in the (1) Ministry of Culture
past, wherein a petition in Hindi was (2) Ministry of Human resource Development
rejected by Supreme Court on the ground
(3) Ministry of Home Affairs
that the language of court was English and
(4) Ministry of Information and Broadcasting
allowing Hindi would be unconstitutional.
ANSWER: 3
53. Which of the following persons may be
represented in the municipality? EXPLANATION:
A. Persons having special knowledge in  The Department of Official Language (Raj
municipal administration. Bhasha Vibhag) comes under the Ministry
of Home Affairs. It was established in June
B. Members of Lok Sabha and state
1975 as an independent department of
legislature assembly representing the
the Ministry of Home Affairs. The
constituencies that comprise the
department issues the annual Programme
municipal area.
and Reports, which are used by Regional
C. Members of Rajya Sabha and the state
Offices and NVS Hqrs to inspect the official
legislative councils registered as electors
language of Regional Offices and JNVs,
within the municipal area.
respectively.
D. The chair persons of the committees’
55. Consider the following statements about the
other than ward committee.
Concurrent list:
Select the correct answer by using the codes
A. If any provision of law made by the State
given below:
Legislature is inconsistent with law made
(1) A and D (2) A, B and D by the Parliament, then the law made by
(3) B and C (4) All of the above the Parliament shall prevail and the law
ANSWER: 4 made by the State Legislature, to the
EXPLANATION: extent of inconsistency, shall be void.

AMIGOS IAS 18
B. If any provision of law made by the State C. Thungon Committee – Recommended
Legislature is inconsistent with law made that the Panchayati Raj bodies should be
by the Parliament, then the law made by constitutionally recognized.
the State Legislature shall prevail, despite Select the correct answer using the code given
the inconsistency, if after been reserved below:
for consideration of the President, has (1) A and C (2) Only B
received his assent. (3) B and C (4) All of the above
Which of the statements given above is/are ANSWER: 1
correct?
EXPLANATION:
(1) Only A (2) Only B
 Balwant Rai Mehta Committee
(3) Both A and B (4) Neither A nor B recommended the establishment of the
ANSWER: 3 scheme of ‘democratic decentralisation’,
EXPLANATION: which ultimately came to be known as
Article 254: Inconsistency between laws made Panchayati Raj.
by Parliament and laws made by the  Ashok Mehta Committee recommended
Legislatures of States that there should be an official
(1) If any provision of a law made by the participation of political parties at all
Legislature of a State is repugnant to any levels of panchayat elections.
provision of a law made by Parliament  Thungon Committee recommended that
which Parliament is competent to enact, the Panchayati Raj bodies should be
or to any provision of an existing law with constitutionally recognized.
respect to one of the matters enumerated 57. Which of the statements given below is
in the Concurrent List, then, subject to the incorrect with respect to Advocate General of
provisions of clause (2), the law made by the State?
Parliament, whether passed before or
(1) He holds office during the pleasure of the
after the law made by the Legislature of
Governor.
such State; shall prevail and the law made
(2) His remuneration is not fixed by the
by the Legislature of the State shall, to the
Constitution.
extent of the repugnancy, be void.
(3) He can quit his office by submitting his
(2) Where a law made by the Legislature of a
resignation to the President.
State with respect to one of the matters
enumerated in the Concurrent List (4) None of the above
contains any provision repugnant to the ANSWER: 3
provisions of an earlier law made by EXPLANATION:
Parliament or an existing law with respect  The term of office of the advocate general
to that matter, then , the law so made by is not fixed by the Constitution. Further,
the Legislature of such State shall, if it has the Constitution does not contain the
been reserved for the consideration of the procedure and grounds for his removal.
President and has received his assent, He holds office during the pleasure of the
prevail in that State: governor. This means that he may be
56. Which of the following Committees and their removed by the governor at any time. He
recommendations is/are correctly matched? may also quit his office by submitting his
A. Balwant Rai Mehta Committee - resignation to the governor.
Recommended the establishment of the Conventionally, he resigns when the
scheme of ‘democratic decentralisation’. government (council of ministers) resigns
B. G.V.K. Rao Committee- Recommended for or is replaced, as he is appointed on its
official participation of political parties at advice. The remuneration of the advocate
all levels of panchayat elections. general is not fixed by the Constitution. He
receives such remuneration as the
governor may determine.

AMIGOS IAS 19
58. Which of the following pairs is/are correctly 59. Which of the following is/are the source(s) of
matched? revenue for the urban local bodies?
A. Indirect elections to the post of A. Entertainment tax
Chairperson of panchayats at the B. Taxes on animals
intermediate and district levels. – C. Rent on municipal properties
Voluntary provision Select the correct answer using the code given
B. Giving representation to the members of below:
the Parliament and the State Legislature in (1) A and C (2) Only C
the panchayats – Compulsory provision
(3) All of the above (4) None of the above
C. Providing reservation of seats for
ANSWER: 3
SC/ST/backward classes in panchayats at
EXPLANATION:
any level. – Compulsory provision
Sources of Municipal revenue are:
Select the correct answer using the code given
below:  Tax Revenue: The revenue from the local
(1) A and B (2) Only B taxes include property tax, entertainment
tax, taxes on advertisements, professional
(3) All of the above (4) None of the above
tax, water tax, tax on animals, lighting tax,
ANSWER: 4 pilgrim tax, market tax, toll on new
EXPLANATION: bridges, octroi and so on. In addition, the
Provisions of the 73rd Amendment Act are: municipal bodies impose various cesses
• Organization of Gram Sabha in a village or like library cess, education cess, beggary
group of villages. cess and so on. Octroi (i.e., taxes on the
• Establishment of panchayats at the entry of goods into a local area for
village, intermediate and district levels. consumption, use or sale therein) has
• Direct elections to all seats in panchayats been abolished in most of the states.
at the village, intermediate and district Property tax is the most important tax
levels. revenue.
• Indirect elections to the post of  Non-Tax Revenue: This source includes
Chairperson of panchayats at the rent on municipal properties, fees and
intermediate and district levels. fines, royalty, profits and dividends,
• 21 years to be the minimum age for interest, user charges and miscellaneous
contesting elections to panchayats. receipts. The user charges (i.e., payment
• Reservation of seats (both members and for public utilities) include water charges,
Chairpersons) for SCs and STs in sanitation charges, sewerage charges and
panchayats at all the three levels. so on.
• Reservation of one-third seats (both 60. Consider the following statements about
members and Chairpersons) for women in Metropolitan Planning Committee (MPC):
panchayats at all the three levels. A. Metropolitan Planning Committee are
• Fixing tenure of five years for panchayats constitutional bodies.
at all levels and holding fresh elections B. Two-thirds of the members of a
within six months in the event of metropolitan planning committee should
supersession of any panchayat. be elected by the elected members of the
• Establishment of a State Election municipalities and chairpersons of the
Commission for conducting elections to panchayats in the metropolitan area from
the panchayats. amongst themselves.
• Constitution of a State Finance C. The plans prepared by Metropolitan
Commission after every five years to Planning Committee have to be
review the financial position of the necessarily enforced by the District
panchayats. Administration.
Which of the statements given above is/are
correct?
AMIGOS IAS 20
(1) Only A (2) A and B prepared by Election Commission of India.
(3) B and C (4) All of the above District Planning committees are formed
ANSWER: 2 under the provisions of 74th Amendment
EXPLANATION: Act.
 243ZE. Committee for Metropolitan  There shall be constituted in every State,
planning. -(I) There shall be constituted in Panchayats at the village, intermediate
every Metropolitan area a Metropolitan and district levels in accordance with the
Planning Committee to prepare a draft provisions of this Part. (2)
development plan for the Metropolitan Notwithstanding anything in clause (1),
area as a whole. Thus, it is a constitutional Panchayats at the intermediate level may
body. The act lays down that two-thirds of not be constituted in a State having a
the members of a metropolitan planning population not exceeding twenty lakhs.
committee should be elected by the Thus, few states which have population
elected members of the municipalities below twenty lakhs do not have
and chairpersons of the panchayats in the intermediate level of Panchayat.
metropolitan area from amongst Arunachal Pradesh recently removed the
themselves. The representation of these intermediate level of Panchayat.
members in the committee should be in 62. Which of the following Commissions
proportion to the ratio between the was/were set up to recommend changes in
population of the municipalities and the the Centre-State relations?
panchayats in that metropolitan area. The A. Verma Commission
chairpersons of such committees shall B. U.C. Banerjee Commission
forward the development plan to the C. Kothari Commission
state government. It is for the state D. Sarkaria Commission
government to implement the plans so Choose the correct code:
prepared by MPC.
(1) Only D (2) A, B and D
61. Consider the following statements:
(3) B, C and D (4) A, B and C
A. District Planning Committee is formed
ANSWER: 1
under the 73rd Amendment Act.
EXPLANATION:
B. According to 73rd amendment act, all
states are to have a three-tiered  The Sarkaria Commission, also known as
panchayati raj structure. the "Commission on Centre-State
Relations", was established in 1983 by the
Which of the statements given above is/are
Indian government. The commission's
not correct?
purpose was to examine the balance of
(1) Only A (2) Only B power between the central and state
(3) Both A and B (4) Neither A nor B governments and suggest changes to the
ANSWER: 3 Constitution of India.
EXPLANATION: 63. Which one of the following statements
 The superintendence, direction and relating to the Comptroller and Auditor
control of the preparation of electoral General (CAG) of India is/are correct?
rolls for, and the conduct of, all elections A. The CAG can attend the sittings of the
to the Panchayats shall be vested in a Committee on Public Accounts.
State Election Commission consisting of a B. The CAG can attend the sittings of Lok
State Election Commissioner to be Sabha and Rajya Sabha.
appointed by the Governor. Thus, the C. The jurisdiction of CAG is co-extensive
elections to panchayats use a separate with powers of the Union Government.
electoral roll which is prepared by State
Select the correct answer using the code given
election commission. This electoral roll is
below:
separate from the one used in General
(1) Only A (2) B and C
and State Legislative Elections which is

AMIGOS IAS 21
(3) A and B (4) All of the above ANSWER: 1
ANSWER: 1 EXPLANATION:
EXPLANATION:  Article 165 of Chapter 2 (The Executive) in
 He acts as a guide, friend and philosopher Part VI (The States) of the Constitution
of the Public Accounts Committee of the deals with the office of the advocate
Parliament. CAG cannot attend the general of the state. This is the only article
sittings of Parliament; it is only attorney dealing with this office.
General of India a non-Parliamentarian  Article 76 of Chapter 1 (The Executive) in
who is allowed to sit in both houses of the Part V (The Union) of the Constitution
Parliament but cannot vote. deals with the office of the Attorney
 The power of CAG is restricted to auditing General of India. This is the only Article
of accounts unlike the Government’s, dealing with this office.
which is responsible from drafting a law to 66. Consider the following statements about
its execution at the ground level. Comptroller and Auditor General of India
64. The Election Commission recognises a A. While dealing with the audit and
political party as a national party if accounting of government companies,
A. It secures at least six percent of the total CAG has certain judicial powers for
valid votes polled in four or more States in prosecuting those who violate the law.
a general election to the Lok Sabha or to B. He audits the accounts related to all
the State Legislative Assemblies. expenditure from the Consolidated Fund
B. If it is recognised as a state party in four of India, consolidated fund of all states but
states. not union territories with legislature.
C. If it wins two percent of seats in the Lok C. He ascertains and certifies the net
Sabha at a general election and these proceeds of any tax or duty, which is final.
candidates are elected from four states. D. He audits the accounts of any other
Select the correct answer using the code given authority when requested by the
below: President or Governor
(1) Only A (2) A and C Which of the following statements given
(3) A and B (4) All of the above above is/are incorrect?
ANSWER: 3 (1) Only B (2) A and B
• A party is recognised as a national party if (3) Only C (4) A, B and D
any of the following conditions is fulfilled: ANSWER: 2
• If it secures six per cent of the valid votes EXPLANATION:
polled in any four or more states at a • CAG audits the accounts related to all
general election the Lok Sabha or to the expenditure from the Consolidated Fund
legislative assembly; and, in addition, it of India, Consolidated fund of each state
wins four seats in the Lok Sabha from any and consolidated fund of each union
state or states. territory having a Legislative Assembly.
• If it wins two per cent of seats in the Lok • CAG is responsible to Parliament only and
Sabha at a general election; and these he is conferred no Judicial powers to
candidates are elected from three states; prosecute anyone.
or • CAG audits the accounts of any other
• If it is recognised as a state party in four authority when requested by the
states. President or Governor.
65. Which among the following is the only Article • He ascertains and certifies the net
in the Indian Constitution that deals with the proceeds of any tax or duty (Art 279). The
office of Advocate general of the state? net proceeds means the proceeds of a tax
(1) 165 (2) 182 or a duty minus the cost of collection.
(3) 147 (4) 195

AMIGOS IAS 22
67. Which of the following statements ANSWER: 4
concerning the Election Commission’s Model EXPLANATION:
Code of Conduct is/are not correct? • Parties form and run governments. The
A. The Model Code of Conduct has been big policy decisions are taken by political
provided by the Constitution of India. executive that comes from the political
B. The Model Code of Conduct emerged out parties. Parties recruit leaders, train them
of a voluntary decision by political parties and then make them ministers to run the
in 1968 to abide by certain norms of government in the way they want.
electoral campaign. • Parties shape public opinion. They raise
C. In case of the violation of the Model Code and highlight issues. Parties have lakhs of
of Conduct, the Supreme Court of India members and activists spread all over the
can step in on its own accord. country.
D. The election Commission of India has the • Parties provide people access to
power to regulate electoral competition government machinery and welfare
by enforcing the Model Code of Conduct schemes implemented by governments.
from the day the dates of the election are For an ordinary citizen it is easy to
announced. approach a local party leader than a
Select the correct answer using the code given government officer.
below: 69. Which of the following statements are true of
(1) A and C (2) Only A the Indian Electoral System?
(3) C and D (4) B and D A. The electoral contest is based on the First
ANSWER: 1 Past the Post system, which ensures
 During the election campaign, the political proportionality of representation.
parties and contesting candidates are B. The Parliament of India has the power to
expected to abide by a Model Code of make laws for the conduct of elections.
Conduct evolved by the Election C. There is a bar to interference by Courts in
Commission on the basis of a consensus electoral matters and no election to the
among political parties. Parliament and State Legislatures may be
 The commission normally announces the questioned except by an election petition.
schedule of elections in major press Select the correct answer using the code given
conference a few weeks before the formal below:
process is set in motion. The Model Code (1) A, B and C (2) All of the above
of Conduct for guidance of candidates and (3) B and C (4) A and C
political parties comes immediately into ANSWER: 3
effect after such announcement. EXPLANATION:
 In case of the violation of the Model Code  First Past the Post electoral system
of Conduct, the Supreme Court of India doesn’t ensure the proportionality of
cannot step in on its own accord. representation i.e., it does not ensure the
68. Which one among the following statements due representation of all minority sections
about the functioning of political parties in a in the society
democracy is not correct?  Article 329 of the Constitution of India,
(1) Political parties give political education to 1950, bars courts from interfering in
the people electoral matters. This means that the
(2) Political parties serve as a link between validity of laws relating to the allotment of
the government and the people seats to constituencies, or the
(3) Political parties fight elections and try to delimitation of constituencies, cannot be
get the maximum number of their questioned in court. Additionally, no
candidates elected election to either House of Parliament or
(4) None of the above to the House or either House of the

AMIGOS IAS 23
Legislature of a State can be questioned the Supreme Court declared this provision
except by an election petition. as unconstitutional on the ground that it
 The Parliament can make laws for seeks to take away the jurisdiction of the
elections to both houses of Parliament Supreme Court and the high courts. It held
and the state legislature. This includes the that the presiding officer, while deciding a
preparation of electoral rolls. question under the Tenth Schedule,
70. Andhra Pradesh Lokayukta Act came into function as a tribunal. Hence, his decision
force on like that of any other tribunal, is subject to
(1) 1st November, 1983 judicial review on the grounds of mala
fides, perversity, etc. But, the court
(2) 2nd October, 1986
rejected the contention that the vesting of
(3) 5th July, 1991 adjudicatory powers in the presiding
(4) 10th January, 1988 officer is by itself invalid on the ground of
ANSWER: 1 political bias.
EXPLANATION: 72. Consider the following statements with
 In Andhra Pradesh, the A.P. Lokayukta Act regard to North Eastern Council:
11 of 1983 came into force with effect A. It was constituted in 1971 by an Act of
from 1-11-1983 and this Institution Parliament.
started functioning from 15-11-1983 and B. It consists of seven states.
since then this Institution has been able to Which of the statements given above is/are
march forward as an effective instrument correct?
in the hands of the public for curbing the
(1) Only A (2) Only B
evil of corruption and maladministration.
(3) Both A and B (4) Neither A nor B
ANSWER: 1
EXPLANATION:
71. Which one of the following statements
regarding disqualification of MPs and MLAs  The North Eastern Council is the nodal
on grounds of defection is not correct? agency for the economic and social
development of the North Eastern Region
(1) These provisions are listed in the Tenth
which consists of the eight States of
Schedules od the Constitution of India.
Arunachal Pradesh, Assam, Manipur,
(2) A member of either House of Parliament Meghalaya, Mizoram, Nagaland, Sikkim
belonging to any political party who is and Tripura. The North Eastern Council
disqualified on the ground of defection was constituted in 1971 by an Act of
shall also be disqualified to be appointed Parliament.
as a minister.
73. Consider the following statements with
(3) Disqualification on grounds of defection regard to the Law Commission of India?
does not apply in case of merger.
A. The first Law Commission in India was
(4) All question regarding disqualification on established under the Charter Act of 1813
grounds of defection are referred to the under the Chairmanship of Lord
presiding officer of the house, whose Macaulay.
decision is final.
B. After India attaining Independence, its
ANSWER: 4 status was upgraded to a Constitutional
EXPLANATION: body with M.C Setalvad as its first
 Any question regarding disqualification chairman.
arising out of defection is to be decided by Which of the statements given above is/are
the presiding officer of the House. correct?
Originally, the act provided that the (1) Only A (2) Only B
decision of the presiding officer is final and
(3) Both A and B (4) Neither A nor B
cannot be questioned in any court.
ANSWER: 4
However, in Kihoto Hollohan case (1993),
EXPLANATION:
AMIGOS IAS 24
 First Law Commission was established in decided by the speaker or the Chairman of
1834 under the Charter Act of 1833 under the concerned House.
the Chairmanship of Lord Macaulay which • Changes in the voting pattern and shift to
recommended codification of the Penal proportional representation of the list
code, the Criminal Procedure Code and a system, instead of present voting system
few other matters. Thereafter, the should be made (However, this matter
second, third and fourth Law Commissions was to be further discussed amongst
were constituted in 1853, 1861 and 1879 exports)
respectively. The first chairman of the Law • There should be fresh delimitation on the
Commission of India was M. C. Setalvad, basis of 1981 census and there should be
who served from 1955–1958. Setalvad a provision for rotation of reserved seats
was also the first Attorney-General of for Scheduled Castes and Scheduled
India. Tribes.
 Law Commission of India is not a • No candidates should be allowed to
constitutional body The first chairman of contest an election from more than two
the Law Commission of India was M. C. constituencies. The age of Candidates for
Setalvad, who served from 1955–1958. assembly seats should be reduced to 21
Setalvad was also the first Attorney- and for the Council to 25.
General of India. • To discourage non-serious candidates, the
74. The Dinesh Goswami Committee security deposit for Lok Sabha should be
recommended increased to Rs. 5000 and for Assembly it
(1) The constitution of state-level election should be increased to Rs. 2500. The
commissioners amount should be forfeited if the
(2) List System of election to the Lok Sabha candidate fails to secure one fourth of the
(3) Governmental funding of parliamentary total votes. The member of proposals to
elections nomination should also be increased.
(4) A ban on the candidature of independent • A model code of conduct be framed which
candidates in the parliamentary elections. would include issues relating to-the use of
ANSWER: 3 official machinery, transport, media,
funds etc.
EXPLANATION:
• There should be a ban on transfer of
• In 1990, Dinesh Goswami Headed a
officials and staff concerned with the
committee made the following
elections. The Commission and the
recommendations;
Central Government should continue the
• The ordering of re-poll or countermanding
periodic revision of election expenses in
should be not only be on the report of the
consulta¬tion with the Election
returning officer, but also otherwise and,
Commission. There should be a six month
also to give the Election Commission the
time limit for holding bye-elections.
requisite powers to appoint investigating
• Army and Para-military personnel,
agencies, prosecuting agencies and
diplomats and others placed outside India
constitution of special courts.
should be allowed proxy voting.
• There is a need for an amendment to the
• Extensive restructuring of the accounting
anti-defection law to restrict
of election expenses is needed.
disqualification only to those cases, where
an elected member voluntarily gives up • Monitoring of expenses should be
his membership of the political party, or undertaken by the Election Commission,
when he votes or abstain from voting and a speedy trial of election disputes
contrary to party whips, directions etc. through the help of adhoc judges should
only in respect of motion of vote of be ensured
confidence. The question of • There should be provision to punish plying
disqualification of members should not be mechanically-propelled vehicles, carrying

AMIGOS IAS 25
lethal weapons and fire arms or Which of the statements given above is/are
distributing liquor on, the polling day. correct?
• Electronic voting machines should be used (1) Only A (2) Only B
to put an end to manipulating and (3) Both A and B (4) Neither A nor B
tempering ANSWER: 1
• Governmental funding of parliamentary EXPLANATION:
elections  A common electoral roll is prepared by
75. Who among the following was the first Election Commission of India for
advocate general of Andhra Pradesh after the conducting elections to both Parliament
formation of the state in 1956? and State Legislatures. However, it is only
(1) E Manohar for the local body elections in each state
(2) V.R. Reddy where a separate electoral roll is prepared
(3) S. Ramachandra Rao by respective State Election Commission
(4) D. Narasa Raju constituted.
ANSWER: 4  Under Article 326 of the Indian
EXPLANATION: Constitution, elections to the Lok Sabha
 D. Narasa Raju was the first advocate and state assemblies to be on the basis of
general of Andhra Pradesh after the adult suffrage.
formation of the state in 1956 78. Consider the following statements about the
Counting and Declaration of electoral result
 Venu Gopal P after the bifurcation of
in India
Andhra Pradesh in 2015.
A. The returning officer is responsible for the
counting of votes and declaration of result
76. Arrange the following committees on
of an election.
Panchayati Raj in chronological order.
B. After the results of all Parliamentary
A. Balwant Rai Mehta Committee
constituencies, the President will
B. Ashok Mehta Committee constitute the new Lok Sabha.
C. LM Singhvi Committee Which of the statements given above is/are
D. GVK Rao Committee correct?
Choose the correct code: (1) Only A (2) Only B
(1) A-B-C-D (2) B-C-A-D (3) Both A and B (4) Neither A nor B
(3) A-B-D-C (4) C-D-A-B ANSWER: 1
ANSWER: 3 EXPLANATION:
EXPLANATION:  According to Sec. 64 of the R. P. Act, 1951,
 Balwant Rai Mehta Committee: 1957 votes are counted by or under the
 Ashok Mehta Committee: 1977 supervision / direction of the Returning
 G V K Rao Committee: 1985 Officer of the Constituency. When the
 L M Singhvi Committee: 1986 counting is completed, the Returning
officer declares the result as per
77. Consider the following statements about
provisions of Sec. 66 of R. P. Act, 1951.
Indian elections:
 According to Sec. 73 of the R. P. Act, 1951,
A. The Constitution provides that elections
after the results of all Parliamentary
to the House of the People and the
constituencies are declared, the Election
Legislative Assemblies of States are to be
Commission will constitute the new Lok
held on the basis of Universal Adult
Sabha by notifying in the official gazette,
suffrage.
the names of the elected members.
B. The Constitution stipulates that there will
79. How many subjects were transferred to
be separate electoral rolls for
Panchayats by the 73rd Constitutional
Parliamentary and State Assembly
Amendment in the Eleventh Schedule?
elections.

AMIGOS IAS 26
(1) 21 (2) 32 C. It lays down standards for legal education
(3) 29 (4) 25 in India.
ANSWER: 3 Which of the statements given above is/are
EXPLANATION: correct?
 The 11th Schedule of the Indian (1) Only C (2) B and C
Constitution was added in 1992 by the (3) All of the above (4) Only A
73rd Constitution Amendment Act. This ANSWER: 3
schedule contains 29 subjects. This EXPLANATION:
schedule covers important topics such as  The Bar Council of India is a statutory body
Panchayat's powers, rural development, created by Parliament to regulate and
poverty alleviation, market, roads and represent the Indian bar.
drinking water, etc.  It performs the regulatory function by
80. Consider the following statements with prescribing standards of professional
regard to National Human Rights conduct and etiquette and by exercising
Commission: disciplinary jurisdiction over the bar. They
A. It can suo motu inquire into any violation also set standards for legal education and
of human rights. grants recognition to universities whose
B. It can look into a matter only within one degree in law will serve as qualification for
year of its occurrence. enrolment as an advocate.
C. It can punish a violator of human rights.  Bar Council of India consists of members
Which of the statements given above is/are elected from each State Bar Council, and
correct? the Attorney General of India and the
(1) A and B (2) B and C Solicitor General of India are ex-officio
(3) All of the above (4) None of the above members.
ANSWER: 1 82. With reference to ‘None of the above’
 Its functions are mainly recommendary in (NOTA) option on electronic voting machines
nature. It has not power to punish the and ballot paper, consider the followings
violators of human rights, nor to award statements:
any relief including monetary relief to the A. Electors were provided the option of
victim. Notably, its recommendations are NOTA in electing both houses of the
not binding on the concerned government Parliament.
or authority. B. In an election if NOTA gets the highest
 The commission is not empowered to votes, then the election will be conducted
inquire into any matter after the expiry of again.
one year from the date on which the act Which of the statements given above is/are
constituting violation of human rights is correct?
alleged to have been committed. In other (1) Only A (2) Only B
words, it can look into matter within one (3) Both A and B (4) Neither A nor B
year of its occurrence. ANSWER: 4
 It can inquire into any violation of human EXPLANATION:
rights or negligence in the prevention of  The voters polled against the NOTA option
such violation by a public servant, either are not taken into account for calculating
suo motu or on a petition presented to it the total valid voters polled by the
or on an order of a court. contesting candidates for the purpose of
81. With respect to the Bar Council of India, return of security deposits to candidates.
Consider the following statements: Even if the number of electors opting for
A. It is a statutory body. NOTA options is more than the number of
B. Attorney General of India is the ex-officio votes polled by any of the candidates, the
member of the Council. candidate who secures the largest

AMIGOS IAS 27
number of votes has to be declared (3) The Prime Minister in consultation with
elected. Chief Minsters of the states.
 NOTA is not applicable to elections (4) None of the above.
conducted for Rajya Sabha and Legislative ANSWER: 1
Councils. EXPLANATION:
83. Under which of the following articles does  The additional functions relating to the
the Rajya Sabha have special powers to services of the Union can be conferred on
legislate a ‘state list' in the national interest? UPSC by the Parliament. It can also place
(1) Article 249 (2) Article 250 the personnel system of any authority,
(3) Article 252 (4) Article 253 corporate body or public institution within
ANSWER: 1 the jurisdiction of the UPSC. Hence the
EXPLANATION: jurisdiction of UPSC can be extended by an
 The correct answer is Article 249. act made by the Parliament.
According to article 249, the Rajya Sabha 86. The jurisdiction of Central Vigilance
has the power to declare that the Commission (CVC) extends to who among the
Parliament has the power to make laws following?
with respect to any matter listed in the A. Group A officers of Central Government
State List in the national interest. B. Officers in Grade D and above in Reserve
Bank of India
84. Which of the following are grounds for C. Members of All India Services
disqualification of a member of Union Public Select the correct answer using the code given
Service Commission? below:
A. If he is adjudged an insolvent. (1) Only A (2) A and B
B. If he engages, during his term of office, in (3) B and C (4) All of the above
any paid employment outside the duties ANSWER: 4
of his office. EXPLANATION:
C. It he is concerned or interested in any The jurisdiction of the CVC extends to the
contract or agreement made by the following:
Government of India. A. Members of All India services serving in
Select the correct answer using the code given connection with the affairs of the Union
below. and Group A officers of the Central
(1) A and B (2) A and C Government.
(3) B and C (4) All of the above B. Officers of the rank of Scale V and above
ANSWER: 4 in the Public Sector Banks.
 The President can remove the chairman or C. Officers in Grade D and above in Reserve
any other member of UPSC from the office Bank of India, NABARD and SIDBI.
under the following circumstances: 87. Which of the following is not a member of the
(a) If he is adjudged an insolvent (that is, has Governing Council of NITI Aayog?
gone bankrupt); (1) Chief Minster of National Capital Territory
(b) If he engages, during his term of office, in of Delhi
any paid employment outside the duties (2) Lieutenant Governor of NCT of Delhi
of his office; or (3) Lieutenant Governor of Andaman and
(c) If he is, in the opinion of the president, Nicobar Islands
unfit to continue in office by reason of (4) Chief Minister of Puducherry
infirmity of mind or body. ANSWER: 2
85. The jurisdiction of the Union Public Service EXPLANATION:
Commission can be extended by The composition of the NITI Aayog is as
(1) An act made by the Parliament follows:
(2) An executive order of the President (a) Chairperson: The Prime Minister of India
AMIGOS IAS 28
(b) Governing Council: It comprises the Chief Which of the following statements given
Ministers of all the State, Chief Ministers above is/are correct?
of Union Territories with Legislatures (i.e, (1) A and B (2) A and C
Delhi, Puducherry and Jammu and (3) Only A (4) All of the above
Kashmir) and Lt. Governors of other Union ANSWER: 4
Territories.
EXPLANATION:
88. Consider the following statements:
 CBI’s primary jurisdiction is confined to
A. Report of the Administrative Reforms Delhi and Union territories. It draws its
Commission, headed by late Morarji power from the Delhi Special Police
Desai, recommended the establishment Establishment (DSPE) Act. The Home
of Lokpal and Lokayukta institutions. Ministry, through a resolution, set up the
B. Lokpal is the highest institution in India to agency in April 1963. Under Section 5 of
investigate Corruption at higher places in the Act, the Central government can
Government. extend its powers and jurisdictions to the
Which of the statements given above is/are states, for investigation of specified
correct? offences.
(1) Only A (2) Only B  The withdrawal of general consent
(3) Both A and B (4) Neither A nor B restricts the CBI from instituting new
ANSWER: 3 cases in the State concerned. However, as
EXPLANATION: decided by the Supreme Court in Kazi
 The Administrative Reforms Commission Lhendup Dorji (1994) the withdrawal of
(ARC) headed by Morarji Desai submitted consent applies prospectively and
an interim report on "Problems of therefore, existing cases will be allowed to
Redressal of Citizen's Grievances" in 1966. reach their logical conclusion.
In this report, ARC recommended the  The Supreme Court made it clear that
creation of two special authorities when it or a High Court directs that a
designated as 'Lokpal' and 'Lokayukta' for particular investigation be handed over to
redress of citizens' grievances. the CBI, there is no need for any consent
 The Lokpal and Lokayuktas Act, 2013, under the DSPE Act.
commonly known as The Lokpal Act, is an 90. The National Investigation Agency is
anti-corruption Act of Indian Parliament in mandated to investigate and prosecute
India which "seeks to provide for the offences under the various Acts mentioned in
establishment of the institution of Lokpal the Schedule of the NIA Act. In this context its
to inquire into allegations of corruption jurisdiction includes investigating cases
against certain important public against which of the following.
functionaries including the Prime Minister A. Offences related to human trafficking.
89. Consider the following statements about B. Counterfeit currency or bank notes.
Central Bureau of investigation: C. Cyber-terrorism.
A. Drawing its powers from Delhi Special D. Manufacture or sale of prohibited arms.
Police Establishment (DSPE) Act, its Choose the correct code:
primary jurisdiction is confined to Delhi (1) A, B and C (2) A, C and D
and Union Territories. (3) B, C and D (4) All of the above
B. The withdrawal of consent to CBI by state ANSWER: 4
governments applies prospectively.
EXPLANATION:
C. When Supreme Court or High Court
 The NIA is empowered to probe terror
directs that a particular investigation be
attacks including bomb blasts, hijacking of
handed over to the CBI, there is no need
aircrafts and ships, attacks on nuclear
for any consent under the Delhi Special
installations and use of weapons of mass
Police Establishment (DSPE) Act.
destruction.

AMIGOS IAS 29
 In 2019, the jurisdiction of the NIA was residuary power to the states. Its other
extended. Consequently, the NIA is also important recommendations are given in
empowered to probe the offences relating the following:
to human trafficking, counterfeit currency • The setting of an Inter-State council
or bank notes, manufacture or sale of immediately
prohibited arms, cyber-terrorism and • Finance commission to be made a
explosive substances. permanent body
91. Consider the following statements about • Deletion of Articles 356, 357, and 365
National Disaster Management Authority which dealt with the President’s rule
A. It consists of a chairperson and other • Abolition of All-India Services (lAS, IPS, and
members, not exceeding nine. IFS)
B. The vice-chairperson has the status of a • Planning Commission to be replaced by a
cabinet minister while the other members statutory body
have the status of a Minister of State. • The Central government completely
Which of the following statements given ignored its recommendations.
above is/are correct? 93. Which one of the following is correct in
(1) Only A (2) Only B respect of the commencement of the election
(3) Both A and B (4) Neither A nor B process in India?
ANSWER: 3 (1) The recommendation for election is made
EXPLANATION: by the government and the notification
 The NDMA consists of a chairperson and for election is issued by the Election
other members, not exceeding nine. The Commission
Prime Minister is the ex-officio (2) The recommendation for election is made
chairperson of the NDMA. The other by the Election Commission and the
members are nominated by the notification for election is issued by the
chairperson of the NDMA. The Home Ministry at the Centre and Home
chairperson of the NDMA designates one Departments in the States
of the members as the vice-chairperson of (3) The recommendation for election is made
the NDMA. The vicechairperson has the by the Election Commission and the
status of a Cabinet Minister while the notification for election is issued by the
other members have the status of a President and Governors of the States
Minister of State. concerned.
92. The abolition of IAS and the I.P.S has been (4) Both the exercises of making a
recommended by the recommendation for election and that of
(1) Dhebar Commission issuing a notification in respect of it are
(2) Kalekar Commission done by Election Commission
(3) Kher Commission ANSWER: 3
(4) Rajamannar Commission EXPLANATION:
ANSWER: 4 • The recommendation for election is made
EXPLANATION: by the Election Commission and the
notification for election is issued by the
• Rajmannar Commission was set up by the
President and Governors of the States
then DMK Government of Tamil Nadu in
concerned.
1969 under the Chairmanship of Dr P.V.
Rajamanar. It was appointed to look into 94. Which one of the following statements
the question regarding the relationship regarding ‘Exit Poll’ is/ correct?
that should subsist between the Centre A. Exit poll is a term used to denote a post-
and the States in a federal set-up. It election survey of voters regarding the
submitted its report in 1971. It demanded candidate in whose favour they had
readjustment of the VII schedule and exercised their franchise.

AMIGOS IAS 30
B. Exit poll and opinion poll are one and the (1) Only A (2) Only B
same. (3) Both A and B (4) Neither A nor B
Which of the statements given above is/are ANSWER: 3
correct? EXPLANATION:
(1) Only A (2) Only B  A party should secure at least six per cent
(3) Both A and B (4) Neither A nor B of valid votes polled in an election to the
ANSWER: 1 state legislative assembly and win at least
EXPLANATION: two seats in that state assembly. A party
 An election exit poll is a poll of voters should secure at least six per cent of valid
taken immediately after they have exited votes polled in an election to Lok Sabha
the polling stations. Unlike an opinion poll, and win at least one seat in Lok Sabha.
which asks for whom the voter plans to 97. Consider the following statements with
vote, or some similar formulation, an exit regard to Discretionary Grants
poll asks for whom the voter actually A. Article 282 empowers both the Centre and
voted. the states to make any grants for any
95. Consider the following statements: public purpose.
A. A Gram Sabha may exercise such powers B. They can only be issued by the Centre and
and perform such functions at the village states for purposes within their legislative
level as the Legislature of a State, may, by competence.
law, provide. Which of the statements given above is/are
B. Parliament may, by law, make provisions correct?
with respect to maintenance of accounts (1) Only A (2) Only B
by Panchayats. (3) Both A and B (4) Neither A nor B
Which of the statements given above is/are ANSWER: 1
correct? EXPLANATION:
(1) Only A (2) Only B  Article 282 empowers both the Centre and
(3) Both A and B (4) Neither A nor B the states to make any grants for any
ANSWER: 1 public purpose, even if it is not within their
EXPLANATION: respective legislative competence. Under
 Gram Sabha may exercise such powers this provision, the Centre makes grants to
and perform such functions at the village the states.
level as the legislature of a State 98. Which of the following are the factors for the
determines. rise of Regional parties in India?
 The State legislature may make provisions A. cultural and ethnic pluralism of the Indian
with respect to the maintenance of society
accounts by the panchayats and the B. failure of national politics to meet the
auditing of such accounts. regional aspirations
96. Which of the following are the conditions for C. absence of a strong opposition party at
recognition as a state party the Central level
A. A party should secure at least six per cent D. alienation and discontentment among the
of valid votes polled in an election to the tribal groups
state legislative assembly and win at least Choose the correct code:
two seats in that state assembly. (1) A, B and C (2) B, C and D
B. A party should secure at least six per cent (3) A, B and D (4) All of the above
of valid votes polled in an election to Lok ANSWER: 4
Sabha and win at least one seat in Lok EXPLANATION:
Sabha.
Other reasons for the rise of regional parties
Which of the statements given above is/are are:
correct?

AMIGOS IAS 31
- economic disparities and regional post inferior to that of a District Judge. The
imbalances in the development constitutional provision enables creation
- desire of certain areas to maintain of the AIJS at District Judge level.
separate identity due to historical factors 101. Who among the following was the first
- self-interest of the deposed maharajas Attorney General of India?
and dispossessed Zamindars (1) G.V. Mavalankar
- reorganization of States on the basis of (2) K.K Venugopal
language (3) M.C. Setalvad
- charismatic personality of the regional (4) None of the above
leaders ANSWER: 3
- factional fights within the larger parties EXPLANATION:
- centralizing tendencies of the Congress • Motilal Chimanlal Setalvad (c. 1884 –
Party 1974) was an eminent Indian jurist, who
- role of caste and religion in the political became the first and longest serving
process Attorney General for India (1950–1963).
99. Consider the following statements regarding 102. Who was the chairman of the first official
the Human Rights Courts language commission?
A. The protection of human rights act, 1993 (1) BG Kher (2) HN Kunzru
provides for the establishment of Human (3) Rajendra Prasad (4) KM Munshi
Rights Court in every district ANSWER: 1
B. They can be set up by the state EXPLANATION:
government only with the concurrence of
• B. G. Kher was the first chairman of the
the chief justice of the high court of that
Official Language Commission, which was
state
established in 1955. Kher was also the first
C. The state government specifies a public chief minister of Bombay after India's
prosecutor or appoints an advocate as a independence.
special public prosecutor for every human
103. How many Lok Sabha seats are reserved for
right court
Scheduled Tribes, as on 26 January 2019?
Which of the statements given above is/are
(1) 47 (2) 54
correct?
(3) 38 (4) 32
(1) A and B (2) A and C
ANSWER: 1
(3) B and C (4) All of the above
EXPLANATION:
ANSWER: 4
• As of January 26, 2019, 47 seats in the Lok
EXPLANATION:
Sabha were reserved for Scheduled
 The human rights courts are set up in Tribes. The Lok Sabha has a total of 131
every district by the state government reserved seats, 84 for Scheduled Castes
only with the concurrence of the chief and 47 for Scheduled Tribes.
justice of the high court of that state for
104. Which of the following subjects is included in
the speedy trial of violation of human the Concurrent List in the Indian
rights.
Constitution?
100. Creation of an All India ‘Services’ is provided A. Forests
in:
B. Banking
(1) Article 324 (2) Article 312
Choose the correct code:
(3) Article 326 (4) Article 310
(1) Only A (2) Only B
ANSWER: 2
(3) Both A and B (4) Neither A nor B
EXPLANATION:
ANSWER: 1
• Article 312 of the Constitution provides
 The Concurrent List is a list of 47 subjects
for the establishment of All India Judicial
that both the Union and State legislatures
Service (AIJS), which shall not include any

AMIGOS IAS 32
have jurisdiction over. The Concurrent List of Lok Sabha and Chief Minister of the
includes subjects of common interest, State concerned.
such as: ANSWER: 4
- Criminal law EXPLANATION:
- Criminal procedure Recommendations with respect to the
- Preventive detention Appointment of the Governor:
- Education - There should be a procedure of consulting
- Forests the Chief Minister of the State, the Vice-
- Weights and measures President of India and the Speaker of the
- Protection of wildlife and birds Lok Sabha in the process of appointing a
Governor.
- Administration of justice
- The State governments must be involved
• Banking is included in the Union List of the
in the appointment of the governor along
Seventh Schedule of the Indian
with the formation of panels.
Constitution.
- Governors must be allowed to complete
105. As per Section 30 of the Protection of Human
five years of their term in office.
Rights Act, 1993 for the purpose of providing
speedy trail of offences arising out of - The individual is required to be an
violation of Human Rights, the State eminent person.
Government may, with the concurrence of - It is recommended that an individual
the Chief Justice of the High Court, by appointed as the Governor must be from
notification specify for each district a outside the state of appointment.
________ to be a Human Rights Court to try - A detached person who has never
the said offences. indulged in active or local politics will be
(1) Judicial Magistrate Court eligible for the post of governor.
(2) Civil Court - The governor, after demitting office, shall
(3) Family Court not be considered eligible for any other
appointment or Office of Profit under the
(4) Court of Session
government.
ANSWER: 4
- Post-retirement benefits must be ensured
EXPLANATION:
to the person who held the position of
• Section 30 of the Protection of Human governor.
Rights Act, 1993 states that a state
107. Which one of the following is not identified
government can designate a Court of
as objective of the Panchayat Extension to
Session as a Human Rights Court (HRC) for
Scheduled Area (PESA) Act?
each district in the state to provide a
(1) To provide self-governance
speedy trial for offenses that arise from
human rights violations. The state (2) To preserve traditional rights
government must get the Chief Justice of (3) To create autonomous regions in tribal
the High Court's concurrence to do this. areas
106. Which one of the following criteria is not (4) To make gram Sabha a nucleus of all
recommended by the ‘Sarkaria Commission’ activities
about the appointment of Governor? ANSWER: 3
(1) He should be eminent in some walk of life EXPLANATION:
(2) He should be a person from outside the The objectives of the Panchayats (Extension to
State the Scheduled Areas) PESA Act are as follows:
(3) He should not be an active politician - To extend the provisions of Part IX of the
(4) Appointment of Governor should be Constitution relating to Panchayats to the
entrusted to a committee comprising the Scheduled Areas with certain
Prime Minister, Home Minister, Speaker modifications.

AMIGOS IAS 33
- To provide self-rule for the bulk of the • Article 243Q of the Constitution of India
tribal population. deals with the constitution of
- To have village governance with municipalities. It gives the Governor the
participatory democracy and to make the power to include or exclude any area, and
Gram Sabha the nucleus of all activities. to constitute a Nagar Panchayat, a
- To evolve a suitable administrative Municipal Council, or a Municipal
framework consistent with traditional Corporation.
practices. 109. Identify the following incorrect pairs about
- To safeguard and preserve the traditions the Election Reform Committee of India and
and customs of tribal communities. its year of establishment:
- To empower Panchayats at the Committee Year
appropriate levels with specific powers A. Santhanam Committee 1964
conducive to tribal requirements. B. Wanchoo Committee 1971
- To prevent Panchayats at the higher level C. Dinesh Goswami Committee 1990
from assuming the powers and authority D. Tarkunde Committee 1974
of Panchayats at the lower level of the Choose the correct code:
Gram Sabha. (1) A and B (2) B and C
108. Consider the following pairs: (3) C and D (4) A and C
Article Provision ANSWER: 1
A. 243 ZE Committee for metropolitan EXPLANATION:
planning • Santhanam – 1962
B. 243 ZD Constitution of Municipalities • Wanchoo – 1970
C. 243 Q Committee for district • Dinesh Goswami – 1990
planning • Tarkunde Committee – 1974
D. 243 ZA Elections to the municipalities 110. The Attorney General of India can appear on
Which of the pairs given above is/are correct? behalf of the Government of India in which of
(1) A and B (2) B and C the following cases in the Supreme Court?
(3) A and D (4) B and D (1) Civil and revenue cases only
ANSWER: 3 (2) Criminal cases only
EXPLANATION: (3) Civil and criminal cases only
• Article 243ZD of the Constitution of India (4) Civil, revenue and criminal cases
establishes a District Planning Committee ANSWER: 4
at the district level in every state. The EXPLANATION:
committee's responsibilities include:
• The Constitution (Article 76) has provided
Consolidating plans from the Panchayats
for the office of the Attorney General for
and Municipalities in the district Preparing
India. He is the highest law officer in the
a draft development plan for the district
country.
• Article 243ZA of the Constitution of India
• The President has assigned the following
gives the State Election Commission the
duties to the Attorney General of India:
power to supervise, direct, and control the
1. To appear on behalf of the Government of
preparation of electoral rolls and the
India in all cases in the Supreme Court in
conduct of all elections to the
which the Government of India is
Municipalities.
concerned.
• Article 243 ZE mandates the constitution
2. To represent the Government of India in
of a Metropolitan Planning Committee
any reference made by the President to
(MPC), for preparing a draft development
the Supreme Court under Article 143 of
plan for metropolitan areas in India that
the Constitution.
have a population of more than 10 lakhs.

AMIGOS IAS 34
3. To appear (when required by the 112. Which of the following Commission/
Government of India) in any High Court in Committees has recommended for the
any case in which the Government of India formation of the post of a Special Officer for
is concerned. Linguistic Minorities?
111. Consider the following statements about the (1) The First Administrative Reforms
recommendations of the Ashok Mehta Commission.
Committee: (2) The Sarkaria Commission.
A. The Panchayati Raj Institutions should (3) The States Reorganization Commission.
have compulsory powers of taxation to (4) The Yugandhar Committee.
mobilise their own financial resources. ANSWER: 3
B. The Nyaya Panchayats should be kept as EXPLANATION:
separate bodies from that of the
• Originally, the Constitution of India did not
Development panchayats.
make any provision with respect to the
Which of the statements given above is/are Special Officer for Linguistic Minorities.
correct? Later, the States Reorganization
(1) Only A (2) Only B Commission (1953-55) made a
(3) Both A and B (4) Neither A nor B recommendation in this regard.
ANSWER: 3 Accordingly, the Seventh Constitutional
EXPLANATION: Amendment Act of 1956 inserted a new
• In 1977, the Janata Government Article 350-B in Part XVII of the
appointed a committee on the Panchayati Constitution.
Raj Institutions, under the chairmanship This Article contains the following provisions:
of Ashok Mehta. It submitted its report in 1. There should be a Special Officer for
1978 and made many recommendations Linguistic Minorities. He is to be appointed
to revive and strengthen the declining by the President of India.
Panchayati Raj system in the country. Its 2. It would be the duty of the Special Officer
main recommendations were: to investigate all matters relating to the
• The three-tier system of Panchayati Raj safeguards provided for linguistic
should be replaced by the two-tier system, minorities under the Constitution.
that is, Zila Parishad at the district level, 3. He would report to the President upon
and below it, the Mandal Panchayat those matters, at such intervals, as the
consisting of a group of villages with a President may direct. The President
total population of 15,000 to 20,000. should place all such reports before each
• A district should be the first point for House of the Parliament and send to the
decentralisation under popular governments of the states concerned.
supervision below the state level. • It must be noted here that the
• The Zila Parishad should be the executive Constitution does not specify the
body and made responsible for planning qualifications, tenure, salaries and
at the district level. allowances, service conditions and
• There should be an official participation of procedure for the removal of the Special
political parties at all levels of the Officer for Linguistic Minorities.
Panchayat elections. 113. The Paris Principles are related to which of
• The Panchayati Raj Institutions should the following bodies:
have compulsory powers of taxation to (1) The Reserve Bank of India
mobilise their own financial resources. (2) The Parliament
• There should be a regular social audit by a (3) The Supreme Court of India
district level agency and by a committee of (4) The National Human Rights Commission
legislators to check whether the funds ANSWER: 4
allotted for the
EXPLANATION:

AMIGOS IAS 35
• The United Nations Paris Principles The Goods and Services Tax Council shall
provide the international benchmarks consist of the following members:
against which the National Human Rights The Union Finance Minister: Chairperson
Institutions (NHRIs) can be accredited by • The Union Minister of State in-charge of
the Global Alliance of National Human Revenue or Finance: Member
Rights Institutions (GANHRI). • The Minister in-charge of Finance or
The Paris Principles set out six main criteria Taxation or any other Minister nominated
that NHRIs require to meet: by each State Government: Member
• Mandate and competence: a broad • The Members of the GST Council shall
mandate, based on universal human rights choose one amongst themselves to be the
norms and standards; Vice-Chairperson of the Council for such
• Autonomy from Government; period as they may decide.
• Independence guaranteed by statute or • Every decision of the GST Council shall be
Constitution; taken at a meeting, by a majority of not
• Pluralism; less than ¾ of the weighted votes of the
• Adequate resources; and members present and voting, in
• Adequate powers of investigation. accordance with the following principles:
114. Who among the following was the first • Vote of the Central Government shall have
Comptroller and Auditor general of India? a weightage of one-third of the total votes
(1) A.K Chanda (2) V. Narahari Rao cast; and
(3) C.K. Daphtary (4) None of the above • Votes of all the State Governments taken
together shall have a weightage of 2/3rd
ANSWER: 2
of the total vote cast.
EXPLANATION:
116. Consider the following statements with
• V. Narahari Rao was the first Comptroller
regard to Advocate General of the state:
and Auditor General (CAG) of India,
A. In the performance of his official duties,
serving from 1948 to 1954. Rao was a civil
the advocate general is entitled to appear
servant in the Indian Audit and Accounts
before any court of law within the state.
Service (IA&AS). In 1954, the Government
of India awarded him the Padma Bhushan, B. He has the right to speak and to take part
the third highest civilian honour, for his in the proceedings of both the Houses of
contributions to civil service. the state legislature.
115. Consider the following statements about the Which of the statements given above is/are
Goods and Services Tax Council (GST Council): correct?
A. The Union Minister of State in charge of (1) Only A (2) Only B
Revenue or Finance is a permanent (3) Both A and B (4) Neither A nor B
member of the GST Council. ANSWER: 3
B. The Vice-Chairman of the GST Council is EXPLANATION:
chosen by the members of the GST Council • In the performance of his official duties,
from one amongst themselves. the advocate general is entitled to appear
C. All decisions in the GST Council shall be before any court of law within the state.
taken by a simple majority of the weighted Further, he has the right to speak and to
votes of the members present and voting. take part in the proceedings of both the
Which of the statements given above is/are Houses of the state legislature or any
correct? committee of the state legislature of
(1) Only A (2) A and B which he may be named a member, but
without a right to vote. He enjoys all the
(3) B and C (4) A and C
privileges and immunities that are
ANSWER: 2
available to a member of the state
EXPLANATION: legislature.

AMIGOS IAS 36
117. Who among the following was the first Chief 120. Which of the following reasons was cited by
Election Commissioner of India? the Ashok Mehta Committee for the decline
(1) Nagendra Singh of Panchayati Raj Institutions in India?
(2) Sukumar Sen A. Role of Bureaucracy
(3) Kalayan Sundaram B. Lack of Political Will
(4) S.L Shakdhar C. Lack of conceptual clarity
ANSWER: 2 D. Lack of gender parity
EXPLANATION: Select the correct answer using the code given
• Sukumar Sen (2 January 1898 – 13 May below:
1963) was an Indian civil servant who was (1) A and D (2) B and C
the 1st Chief Election Commissioner of (3) A, B and C (4) B, C and D
India, serving from 21 March 1950 to 19 ANSWER: 3
December 1958. EXPLANATION:
118. Article 76 of the Indian Constitution deals The committee cited the following reasons:
with which of the following offices? • Role of Bureaucracy
A. Attorney General of India • Lack of political will
B. Solicitor General of India • Lack of conceptual clarity
C. Additional Solicitor General of India • Haphazard programs
Choose the correct code: • Non-performance
(1) A and C (2) A and B • Rise of vested interests
(3) B and C (4) Only A • Lack of adequate financial resources
ANSWER: 4 121. Information under the RTI Act, 2005 can be
EXPLANATION: provided in respect of
 In addition to the AG, there are other law (1) National Security Council Secretariat
officers of the Government of India. They (2) Assam Rifles
are the solicitor general of India and (3) Border Road Development Board
additional solicitor general of India. They
(4) Border Road Organisation
assist the AG in the fulfilment of his official
responsibilities. It should be noted here ANSWER: 3
that only the office of the AG is created by EXPLANATION;
the Constitution. In other words, Article  The Border Roads Development Board
76 does not mention about the solicitor (BRDB) is listed in Schedule II of the RTI
general and additional solicitor general. Act, 2005. However, the Border Roads
119. The office of Lokpal and Lokayukta in India is Organisation (BRO) is not listed and is
based on which one of the following? considered exempt from the RTI Act,
(1) Parliamentary Commissioner of UK 2005.
(2) Ombudsman in Scandinavia  There are 26 intelligence and security
(3) Procurator General of Russia organisations established by the Central
government such as the Intelligence
(4) Council of State in France
Bureau, Research and Analysis Wing,
ANSWER: 2 Directorate of Enforcement, National
EXPLANATION: Technical Research Organisation, National
 The office of Lokpal and Lokayukta in India Security Council Secretariat that are
is based on the ombudsman. The Lokpal exempt under RTI.
and Lokayukta Act of 2013 established the 122. 73rd Constitutional Amendment is not
Lokpal for the Union and Lokayukta for the applicable to which among the following
states. states?
A. Meghalaya
B. Mizoram

AMIGOS IAS 37
C. Nagaland  Communist Party of India- Marxist was
Choose the correct code: formed in 1964.
(1) A and B (2) B and C 125. Which one of the following is NOT a classified
(3) A and C (4) A, B and C category of political parties as outlined by the
ANSWER: 4 Election Commission of India?
EXPLANATION: (1) National Parties
 The 73rd Constitutional Amendment Act (2) State Recognised Parties
does not apply to the states of Nagaland, (3) Regional Parties
Meghalaya and Mizoram and certain (4) Registered Unrecognised Parties
other areas. The other areas include, (a) ANSWER: 3
the scheduled areas and the tribal areas in EXPLANATION:
the states; (b) the hill areas of Manipur for  Classification of political parties – (1) For
which district councils exist; and (c) the purposes of this Order and for such
Darjeeling district of West Bengal for other purposes as the Commission may
which Darjeeling Gorkha Hill Council specify as and when necessity therefor
exists. arises, political parties are either
123. In which year "None of the above" (NOTA) recognised political parties or
button was introduced in Electronic Voting unrecognised political parties. (2) A
Machines recognised political party shall either be a
(1) 2009 (2) 2014 National party or a State party.
(3) 2011 (4) 2013 126. A constitutional government by definition is
ANSWER: 4 a
EXPLANATION: (1) Government by legislature
 Election Commission of India had (2) Popular government
introduced ‘NOTA’ – ‘None of the Above’ (3) Multi-party government
option on ballot papers displayed on (4) Limited Government
EVMs and all other ballot papers with ANSWER: 4
effect from 11th October, 2013, pursuant EXPLANATION:
to Supreme Court’s Order dated 27th
 Constitutional government is by definition
September, 2013.
limited government. It means government
124. Consider the following pairs with regard to conducted according to rules and
formation of political parties in India: principles, which are binding on all
Political Party Year of formation political actors, and which therefore help
A. All India Trinamool to constrain the unfettered exercise of
Congress 1998 power by separating or dividing it.
B. Bahujan Samaj Party 1984 127. Under which Schedule of the Constitution of
C. Communist Party of India can the transfer of tribal land to private
India – Marxist 1964 parties for mining be declared null and void?
Choose the correct code: (1) Third Schedule (2) Fifth Schedule
(1) A and C (2) B and C (3) Ninth Schedule (4) Twelfth Schedule
(3) A and B (4) All of the above ANSWER: 2
ANSWER: 4 EXPLANATION:
EXPLANATION:  According to the Samatha v. State of
 All India Trinamool Congress (AITC): Andhra Pradesh & Ors (1997) judgment
Launched on 1st January 1998 under the where the Supreme Court declared that
leadership of Mamata Banerjee. the transfer of tribal land to private
parties for mining was null and void under
 Bahujan Samaj Party (BSP): Formed in
the Fifth Schedule.
1984 under the leadership of Kanshi Ram.

AMIGOS IAS 38
128. M.M. Punchhi Commission on Centre-State officer for every polling booth in the
Relations has recommended the disposal of a constituency.
bill reserved for the consideration of the 130. Which among the following articles need to
Union Executive within be amended for conducting simultaneous
(1) Four months (2) Six months elections to Parliament and State Legislative
(3) Eight months (4) Five months Assemblies in India?
ANSWER: 2 A. Article 172 B. Article 83
EXPLANATION: C. Article 356 D. Article 87
 The M.M. Punchhi Commission on Centre- E. Article 174
State Relations made a recommendation Choose the correct code:
regarding the disposal of a bill reserved for (1) A, C and E (2) B, D and E
the consideration of the Union Executive. (3) A, B, C and E (4) A, B and E
This means that according to the ANSWER: 3
recommendation made by the Punchhi EXPLANATION:
Commission, the Union Executive should
 Following articles need to be amended for
dispose of a bill within a period of six
conducting simultaneous elections to
months.
Parliament and State Legislative
129. Consider the following statements
Assemblies in India -
A. The election commission is assisted by
 Article 83 – Duration of Houses of
Deputy Election Commissioners who are
Parliament.
drawn from Civil Service and appointed by
the commission with tenure system  Article 85 – Dissolution of the House of the
People by the President.
B. At the state level, the election commission
is assisted by the Chief Electoral Officer  Article 172 – Duration of the State
who is appointed by the president Legislatures.
C. At the district level, the collector acts as  Article 174 – Dissolution of the State
the district returning officer Legislatures.
Which of the statements given above is/are  Article 356 – Imposition of the President’s
correct? rule in the States.
(1) A and C (2) B and C 131. Consider the following statements with
(3) A and B (4) All of the above regard to Electoral Bonds Scheme 2018:
ANSWER: 1 A. Only those political parties registered
under section 29A of the Representation
 The election commission is assisted by
of People Act, 1951 and secured not less
Deputy Election Commissioners who are
than one per cent of the votes polled in
drawn from Civil Service and appointed by
the last General Election to the House of
the commission with tenure system. They
the People or the Legislative Assembly are
are assisted, in turn, by the secretaries,
eligible to receive funds through electoral
joint secretaries, deputy secretaries and
bonds.
undersecretaries posted in the secretariat
of the commission B. They will be valid for a period of fifteen
days from the date of issue.
 At the state level, the election commission
C. All transactions involving an electoral
is assisted by the Chief Electoral Officer
bond will take place through the verified
who is appointed by the Chief Election
account that the Election Commission of
Commissioner in consultation with the
India (ECI) will assign to the party.
state government.
D. The donor’s name will not appear on the
 At the district level, the collector acts as
electoral bonds.
the district returning officer. He appoints
the returning officer for every Which of the statements given above is/are
constituency in the district and presiding correct?
(1) A, B and D (2) B, C and D

AMIGOS IAS 39
(3) B and C (4) All of the above (4) Punchhi Commission
ANSWER: 4 ANSWER: 2
EXPLANATION: EXPLANATION:
 Any party that has obtained at least 1% of  The Justice Thakkar Commission of Inquiry
the votes cast in the most recent general was established to investigate the
election or assembly election and is circumstances leading to the assassination
registered under section 29A of the of Prime Minister Indira Gandhi. The
Representation of the Peoples Act, 1951 commission was led by Justice Manharlal
(43 of 1951), is eligible to receive electoral Pranlal Thakkar. The report recommended
bonds. All transactions involving an a separate investigation into the
electoral bond will take place through the conspiracy angle behind the assassination.
verified account that the Election 134. Under the RTI Act, 2005, an applicant making
Commission of India (ECI) will assign to the request for information shall be required to
party. give:
 The donor’s name will not appear on the A. His contact details
electoral bonds. Therefore, it’s possible B. His personal details
that the political party is unaware of the C. Reasons for requesting the information
participant’s identity. Select the correct answer using the code given
 The Electoral Bonds shall be valid for below:
Fifteen Calendar Days from the Date of (1) Only A (2) B and C
Issue ie: An Electoral Bond issued on 1st
(3) A and B (4) All of the above
March 2018 will be valid upto 15thMarch
ANSWER: 1
2018.
EXPLANATION:
 The Electoral Bonds can be redeemed only
by an eligible Political Party by depositing  According to the Right to Information Act,
the same in their Designated Bank 2005, an applicant requesting information
Account maintained with Authorised is only required to provide their contact
Bank. details. They are not required to provide
any other personal details or reasons for
requesting the information.
132. Which one of the following is not a level in
the Panchayati Raj structure in India? 135. The Judgment of the Supreme Court in
Peoples' Union for Civil Liberties v. Union of
(1) Gram Panchayat
India 2004 is related to which of the
(2) Block Samiti following?
(3) Mandal Committee (1) Right to shelter
(4) Zila Parishad (2) Right against custodial violence
ANSWER: 3 (3) Right to Information
EXPLANATION: (4) Right to speedy trail
The Panchayati Raj system in India has three ANSWER: 3
levels:
EXPLANATION:
• Village level: Gram Panchayat
 The Court observed that “the right to hold
• Block level: Panchayat Samiti or Janpad a telephone conversation in the privacy of
Panchayat one's home or office without interference
• District level: District Parishad can certainly be claimed as 'right to
133. Which one of the following Commissions has privacy'” and held that telephone-tapping
not examined the issue of removal of the would violate Article 21 unless it was
Governor of a State? permitted under a “procedure established
(1) Sarkaria Commission by law”.
(2) Thakkar Commission 136. Which of the following Amendment Acts
(3) Venkatachailiah Commission lowered the voting age of elections to the Lok

AMIGOS IAS 40
Sabha and to the Legislative Assemblies of • Shri. Ajay Narayan Jha, former member,
States from 21 years to 18 years? 15th Finance Commission and former
(1) The 58th Constitutional Amendment Act Secretary, Expenditure
(2) The 61st Constitutional Amendment Act • Smt. Annie George Mathew, former
(3) The 63rd Constitutional Amendment Act Special Secretary, Expenditure
(4) The 71st Constitutional Amendment Act • Dr. Niranjan Rajadhyaksha, Executive
ANSWER: 2 Director, Artha Global
EXPLANATION: • Dr. Soumya Kanti Ghosh, Group Chief
Economic Advisor, State Bank of India
 The Sixty-first Amendment of the
Constitution of India, officially known as 139. Which article of the Indian Constitution deals
The Constitution (Sixty-first Amendment) with Finance Commission of India?
Act, 1988, lowered the voting age of (1) 260 (2) 280
elections to the Lok Sabha and to the (3) 255 (4) 244
Legislative Assemblies of States from 21 ANSWER: 2
years to 18 years. This was done by EXPLANATION:
amending Article 326 of the Constitution,  Article 280 of the Constitution of India
which concerns elections to the Lok Sabha establishes the Finance Commission. The
and the Assemblies. President is responsible for appointing the
137. Who among the following was the chairman Commission. The Commission's purpose is
of the first finance commission appointed in to provide recommendations on the
1951? distribution of tax revenues between the
(1) A.K Chanda (2) K.C. Neogy Union and the States, and among the
(3) K. Santhanam (4) Y.B Chavan States themselves.
ANSWER: 2 140. ‘Community Development Programme’ was
EXPLANATION: started in which of the following years in
 K.C. Neogy was the chairman of the first India?
Finance Commission of India, which was (1) 1955 (2) 1952
appointed in 1951. The commission was (3) 1956 (4) 1948
established by Dr. B.R. Ambedkar, the ANSWER: 2
then law minister, to address imbalances EXPLANATION:
between the Centre and the States. The • Community development in India was
commission's term was from 1952–1957. initiated by Government of India through
138. Who among the following were members of Community Development Programme
Sixteenth Finance Commission? (CDP) in 1952. The focus of CDP was on
A. Anni Goerge Mathew rural communities. But, professionally
B. Niranjan Rajadhyaksha trained social workers concentrated their
C. Arvind Panagariya practice in urban areas.
Choose the correct code: 141. Consider the following statements with
(1) A and C (2) C only regard to statutory grants:
(3) B and C (4) All of the above A. Article 275 empowers the Parliament to
make grants to the states which are in
ANSWER: 4
need of financial assistance.
EXPLANATION:
B. They are charged on the consolidated
 The Sixteenth Finance Commission was fund of India every year.
constituted on 31.12.2023 with Shri
Which of the statements given above is/are
Arvind Panagariya, former Vice-Chairman,
correct?
NITI Aayog as its Chairman.
(1) Only A (2) Only B
Other full time members include-
(3) Both A and B (4) Neither A nor B
ANSWER: 3

AMIGOS IAS 41
EXPLANATION: ANSWER: 1
• Article 275 empowers the Parliament to EXPLANATION:
make grants to the states which are in • The Second Administrative Reforms
need of financial assistance and not to Commission (ARC) was constituted on
every state. Also, different sums may be 31.08,2005, as a Commission of Inquiry,
fixed for different states. These sums are under the Chairmanship of Shri Veerappa
charged on the Consolidated Fund of India Moily for preparing a detailed blueprint
every year. Apart from this general for revamping the public administrative
provision, the Constitution also provides system.
for specific grants for promoting the 144. Which among the following is the first inter-
welfare of the scheduled tribes in a state state water disputes tribunal formed in
or for raising the level of administration of India?
the scheduled areas in a state including (1) Godavari Water Disputes Tribunal
the State of Assam. The statutory grants
(2) Narmada Water Disputes Tribunal
under Article 275 (both general and
(3) Cauvery Water Disputes Tribunal
specific) are given to the states on the
recommendation of the Finance (4) Krishna Water Disputes Tribunal - I
Commission. ANSWER: 4
142. Consider the following statements: EXPLANATION:
A. A state cannot raise any loan without the • The Krishna Water Disputes Tribunal
consent of the Centre, if there is still (KWDT) was the first inter-state water
outstanding any part of a loan made to the disputes tribunal in India, established in
state by the Centre. 1969. The tribunal was chaired by R.S.
B. The Central government can make loans Bachawat and involved the states of
to any state or give guarantees in respect Karnataka, Andhra Pradesh, and
of loans raised by any state. Maharashtra.
Which of the statements given above is/are 145. Which of the items below does not fall in the
correct? State list?
(1) Only A (2) Only B (1) Higher education
(3) Both A and B (4) Neither A nor B (2) Public health
ANSWER: 3 (3) Agriculture
EXPLANATION: (4) Land and water
• The Central government can make loans ANSWER: 1
to any state or give guarantees in respect EXPLANATION:
of loans raised by any state. Any sums • Higher education in India is a joint
required for the purpose of making such responsibility of the states and the central
loans are to be charged on the government. The Indian constitution's
Consolidated Fund of India. Seventh Schedule includes three lists:
• A state cannot raise any loan without the Union, State, and Concurrent. Before
consent of the Centre, if there is still 1976, education was the sole subject of
outstanding any part of a loan made to the the states, with the central government
state by the Centre or in respect of which only concerned with standards and
a guarantee has been given by the Centre. coordination of higher and technical
143. Who among the following was the chairman education. The 42nd amendment to the
of the 2nd Administrative Reforms Constitution of India in 1976 placed
Commission? education in the Concurrent list, allowing
the central government to legislate on it.
(1) Shri Veerappa Moily
146. Which one among the following political
(2) Moraji Desai
parties came second in terms of seats won in
(3) K Hanumanthayya
the first Lok Sabha election in India?
(4) M.C. Setalvad

AMIGOS IAS 42
(1) Kisan Mazdoor Praja Party C. Southern Zonal Council Chennai
(2) Communist Party of India D. Eastern Zonal Council Kolkata
(3) People’s Democratic Front Which of the pairs given above is/are correctly
(4) Socialist Party matched?
ANSWER: 2 (1) A and D (2) A, B and C
EXPLANATION: (3) A, C and D (4) All of the above
• The Indian National Congress (INC) won ANSWER: 4
364 seats. They were followed by EXPLANATION:
Independents, winning a total of 37 seats.
The Communist Party of India (CPI) and
the Socialist Party (India) followed with 16
and 12 seats respectively.
147. First democratically elected communist party
government formed in which of the following
states in India?
(1) Tamil Nadu (2) Manipur
(3) Kerala (4) West Bengal
ANSWER: 3
EXPLANATION:
• The first democratically elected
communist government in India was 150. Who among the following is the chairman of
formed in Kerala in 1957, when the North Eastern Council?
Communist Party of India (CPI) won the (1) Minister of Development of North Eastern
state elections. The CPI became the Region
largest opposition party in the general
(2) President
elections of 1957, and E. M. S.
(3) Union Home Minister
Namboodiripad became the first chief
minister of Kerala. The government was (4) Prime Minister
formed with the support of five ANSWER: 3
independent legislators, but was unable EXPLANATION:
to complete its full five-year term. • The Union Home Minister is ex-Officio
148. Article 356 was used for the first time in Chairman and the Minister DoNER is the
which of the following states? ex-Officio Vice-Chairman of the NEC.
(1) Tamil Nadu (2) Pubjab
(3) Karnataka (4) Assam
ANSWER: 2
EXPLANATION:
• The state of Punjab was the first state to
have a state emergency declared in 1951.
The state emergency was declared under
Article 356, also known as President's
Rule. The article allows the president to
impose presidential rule in any state
under the recommendation of the state's
governor.
149. Consider the following pairs:
Zonal Council Headquarters
A. Central Zonal Council Allahabad
B. Western Zonal Council Mumbai

AMIGOS IAS 43

You might also like